You are on page 1of 68

6.

In performing mandibular nerve block for a


child patient, the most probable cause of failure
of anaesthesia is? (AI 2009)
l. Primary teeth are lighter than permanent teeth A. More concentration of L A is required
because: (AJIMS No v 08)
8. Because of different angulation of the needle
A. Thin dentin in primary teeth C. Because of position of mandibular foramens
B. Difference in crystalline structure makes
D. None of the above
refractive index different
C. Because of contrast with the adjacent tissue
7. During administration of NSAIDS in children
D. Lighter color of enamel
the most important of dose determination is?
A. Weight of the child (AI 2009)
1. Cat eye syndrome is due to: (AI 2009)
B. Age of the child
A. Partial trisomy 18
C. Nature of the drug
B. Partial trisomy 13
D. Chronicity of pain
C. Partial trisomy 21
D. Partial trisomy 22
8. A child has to be given pre medication for
behaviour modilication, which is the most
3. The most common cause of short stature is? preferred route? (AI 2009)
A. Constitutional delay in growth (AI 2009) A. Oral
8 . Systemic diseases 8. Intravenous
C. Hypothyroidism C. Intramuscular
D. Growth hormone deficiency D. lntramucosal

4. Most common mode of treatment of a 1 year old 9. The preferred roote for anaesthesia for a child
child with asthma is (AI 2009) patient are aU except? (AI 2009)
A. Inhaled short acting P2 - agonist A. Oral
B. Oral short acting theophylline 8. Intravenous
C. Oral Ketolifen C. Submucosal
D. Leukotriene agonist D. Intradermal

S. An infant with cleft lip, cleft palate, polydactyly, 10. Defmition of Autistic child is? (AI 2009)
microcephaly with holoprosencephaly,
A. Incapacitating communication
ectodermal scalp defect is suffering from?
8. Problem in learning
A. Trisomy 21 (AI 2009)
C. Incapacitating communication and emotional
B. Trisomy 18
problems
C. Trisomy 13
D. None of the above
D. Turner syndrome
452. Dentogist: MCQs in Dentistry - Clinical Sciences

11. All are seen n a cbild witb cerebral palsy, 18. Tbere are two normal children of similar age
except? (AI 2009) and sex. In one cbild all permanent teetb except
A. Fluorosis 3r d molars bave erupted, in the other no
B. Increased caries permanent canine and I" premolar are there,
C. Increased salivation Tbere ages are: . (AI 2001)
D. Trauma A. 7 years
B. 9 years
12. Premature synostoses of coronal suture along C. 11 years
witb basal sutures sbows wbicb of tbe following D. 13 years
features (AIIMS May 2008)
A. Brachycephaly 19. Tbe correct dentition in a 9 year old child is:
B. Oxycephaly A. 12CDE6 (AI 2001)
C. Trigonocephaly B. 12C456
D. Scaphocephaly C. 123DE6
D. 123456
13. The disorder characterized by craniosynostoses,
craniofacial anomalies, severe symmetrical 20. Into bow many segments the infant's gum pad
syndactyly (cutaneous and bony fusion) of is divided: (AI 2004)
bands and feets along witb preaxial A. two in each quadrant
polysyndactyly and variable soft form B. three in each quadrant
syndactyly (AIIMS May 2008) C. two in each jaw
A. Carpenter syndrome D. five in each quadrant
B. Crouzon syndrome
C. Apert syndrome 21. Tbe seizure witb no tonic or clonic seizure tbat
D. Down's syndrome occurs asymptomatic in children with several
times a day is? (AIIMS May 2007)
A. Grandmal epilepsy
14. Wbicb of the following sbows lowest frequency
B. Petitmal epilepsy
of cleft lip and palate. (AIIMS May 2008)
C. Infantile epilepsy
A. Negroes
D. Tonic- clonic epilepsy
B. Afghanese
C. American Indian
22. Untrue about difference between primary and
D. Indian permanent teeth is? (PGI Dec 2006)
A. Dentin is thicker in primary teeth
15. The deciduous teeth can be differentiated from B. Pulp chambers are wide in primary teeth
permanent teetb by? (AIIMS Nov 2006)
C. Enamel thickness is less in primary teeth
A. Enamel prismatic arrangement D. Pulp horns are high.
B. Dentinal tubule arrangement
C. Primary and secondary dentin 23. Cbronological Age is defined as? (PGI Dec 2006)
D. Mineral content A. It is the age from the birth of the child.
B. Age of physiological development
16. A 5-year-old boy passed 18 loose stools in last C. Age of skeletal maturation.
24 hours and vomited twice in last 4 hours. He D. All of the above
is irritable but drinking fluids. The optional
tberapy for this child is? (AIIMS May 2006) 24. Which of the following Oropbaryngeal function
A. Intravenous fluids is not involuntary? .
B. Oral rehydration therapy A. Tongue position
C. Intravenous fluid initially for 4 hours followed B. Respiration
by oral fluids . C. Gagging
D. Plain water and lime D. Speech

17. Kalu, a 6 year old cbild, receive tetracycline. 25. The fear of a 6-year-oldrelated to dentistry is
Noticeable discolourtion will be seen in: primarily: (AI 2007)
A. Premolars, incisors and lst molars A. Subjective
B. Canine and 2nd molars (AI200l) B. Objective
C. Canines, premolars and 2nd molars C. Subjective and objective
D. Incisors and l" molars D. Psychological
454. Dentogist: MCQs in Dentistry - CLinicaL Sciences

42. In a spaced primary dentition with a straight 50. Tooth buds generally initiated after birth are:
terminal plane a class I permanent molar rela- A. Entire permanent dentition only
tionship is achieved by: B. All permanent and some primary teeth
A. Immediate eruption into a class I relationship C. First and second premolars and second and
B. Early mesial shift third molar only
C. Late mesial shift D. It is very variable
D. Forward jaw thrust

43. There is a relative decrease in arch length by aU


of the following except:
A. Eruption of the permanent molars 51. Which surface of primary first molar is most
B. Exfoliation of the primary molars susceptible to dental caries? (AI/MS 2008)
C. Proximal caries in primary teeth A. Proximal surface above the contact area
D. Eruption of the permanent incisors B. Proximal surface below the contact area
C. Occlusal surface in pits and fissures
44. When the mandibular lateral incisors erupt D. Buccal surface below the height of contour
they bring about:
A. An increase in arch length 52. Pulpal infection in primary mandibular
B. A decrease in arch length posterior teeth is first manifested on a
C. An increase in overbite radiograph in the area?
D. A decrease in overbite A. Of furcation
B. At the apex of the root
45. The lingual eruption of the lower permanent C. Around the permanent tooth bud
central incisors with regard to the milk counter- D. Near the crest of alveolar bone
parts is corrected by:
A. Simple lower plate appliance '53. The "Pulse oximetry" used in the determination
B. Fixed orthodontic appliance of? (AI/MS May 2006)
C. Exercises of the oral musculature A. Rate of flow
D. Self-correcting anomaly B. Oxygen saturation
C. Blood volume
46. The deformity of cleft palate occurs during D. Blood coefficient
which trimester of pregnancy?
A. First 54. The Buckley's solution is composed of?
B. Second (AIIMS May 2006)
C. Third A. Cresol, formaldehyde, water and glycerin
D. Any of the above B. Formaldehyde, resorcinol, water
C. Iodoform, glutaraldehyde, ZOE
47. The most common orofacial deformity that pro- D. Chloraldehyde, parachloral, methanol water
duces malocclusion is:
A. Osteogenesis imperfecta 55. The KRI paste is composed of?
B. Cleidocranial dysostosis A. Iodoform, camphor, parachlorophenol and
C. Cleft palate menthol . (AI/MS May 2006)
D. Pierre Robin syndrome B. Iodoform and ZOE
C. Paracholorophenol, camphor and menthol
48. In the absence of a second premolar the roots of D. Calcium hydroxide and iodoform
the primary second molar will most likely:
A. Resorb at normal site 56. In the proximal portion of a deeply extending
B. Resorb more slowly than normal Class II cavitypreparation in a deciduous
C. Resorb more rapidly than normal mandibular 20 molar the gingival seat cannot
D. Show no resorption as the initiator force is be obtained properly because: (AI 2001)
absent .A. The enamel rods are present in the gingival
third
49. Ectodermal cells are responsible for formation B. Buccal and gingival surface of the deciduous
of: molar converge occlusally
A. Alveolar bone C. The primary teeth have a pronounced cervical
B. Periodontal tissue constriction
C. Cementum formation D. The proximal surface of the deciduous 20d
D. Determining cro~ and root shape molar is broad and flat
Questions _ 455

57. The facial and lingual walls of the proximal box B. Root canal therapy
in case of Class II amalgam filling for a primary C. Calcium hydroxide pulpotomy
tooth are: (AI 2001) D. Apexification
A. Parallel
B. Diverge occlusally 64. The best filling material in deciduous teeth for
C. Convex RCT: (AI 2002)
D. Follow the direction of the enamel rods A. Gutta-percha
B. Calcium hydroxide
58. Babu, a 10 year old child, has a mid crown
C. Zinc Oxide Eugenol
fracture of maxillary central incisor. The
D. Amalgam
treatment of choice is: (AI 2001)
A. Formacresol pulpotomy
B. Conventional root canal treatment 65. The most important morphologic or histologic
C. Calcium hydroxide pulpotomy consideration in cavity preparation in primary
D.. Will depend on radiographic findings teeth is: (AI 2003)
A. Size of primary molars
59. Which pulp horn of the deciduous mandibular B. Thickness of enamel and dentin
2Dd molar be exposed during Class II cavity C. Direction of roots below CEJ
preparation: (AI 2001) D. Direction of enamel rods cervically
A. Mesiobuccal
B. Mesiolingual 66. Which of the following is the most
C. Distolingual recommended endodontic obturating material
D. Distobuccal for deciduous teeth: (AI 2004)
A. Kri paste
60. Babu, a 71/2 year old child, reprts with a B. ZOE
fracture of central incisor with an open apex, C. Ca(OH)2 paste
there is large pulpal exposure. The treatment of D. Maisto's paste
choice is: (AI 2001)
A. Smooth the edges and protect the exposed
67. Which of the following have obtundant effect on
portion with ZOE dressing
the pulp? (AIIMS May 2007)
B. Pulpectomy and calcium hydroxide dressing
A. Zinc oxide eugenol
C. Pulpotomy and calcium hydroxide dressing
B. Calcium hydroxide
D. Direct pulp capping with calcium hydroxide
dressing C. Glass ionomer cement
D. Zinc phosphate
61. On the next day following formocresol
pulptomy in the 2nd deciduous molar of a 3 year 68. The "PULP OXIMETRY" used in the
old child, an ulcer is noticed in the midline of determination of? (AIIMS Nov 2007)
the lower lip. The most common cause is: A. Rate of flow
A. Lip biting (AI 2001) B. 02 saturation
B. Allergic reaction to local anesthesia C. Blood volume
C. Leakage of formocresol D. Blood coefficient
D. Pressure injury due to rubber dam application
69. MTA is used for all except? (PGI Dec 2006)
62. The purpose of pulpotomy is (AI 2002) A. Pulpotomy procedures
A. To preserve the coronal pulp B. Apexification of immature teeth
B. To preserve the vitality ofrdicular pulp
C. Perforation of tooth
C. To prevent the pulp exposure while removing
D. Restoration of Class II, III cases.
carious dentin
D. None of the above
70. Primary function ofMTA is? (PGI Dec 2006)
63. A child has mid crown fracture in a permanent A. Root canal restorative material in deciduous
maxillary central incisor 'several months ago, teeth.
the pulp is now necrotic, radiographic B. Restoration in deciduous anterior teeth.
examination reveals an incomplete' formed root C. Apexification in an immature traumatized
with an open apex, treatment of choice is: pulpless teeth.
A. Formocresol pulpotomy (AI 2002) D. None
456 • Dentogist: MCQs in Dentistry - Clinical Sciences

71. Untrue about Cariosolv is? (PGI Dec 2006) B. Zinc phosphate cement
A. It is chemico- mechanical men of caries C. ZOE
removal D. Glass ionomer cement
B. Contain sodium hypochlorite.
C. Has a pH of7.0 80. Ellis Class III # of tooth involves:
D. Has leucine, lysine, glutamic acid as its A. Enamel
component B. Enamel and dentin
C. Enamel, dentin and pulp
72. Cariosolv is used for? (PGI June 2007) D. Crown and root
A. Staining carious dentin
B. Softening of dentin for removal of carious part 81. The simplest form of pulp therapy is:
C. Enamel caries detection A. Apicoectomy
D. Study histologic section of tooth B. Pulp capping
C. Pulpectomy
73. For Stainless steel crown the tooth surface D. Pulpotomy
requiring least amount of reduction is?
A. Buccal (PGI June 2008) 82. Class II cavity preparation for amalgam resto-
B. Lingual ration in deciduous teeth requires: (AI 2005)
C. Occlusal A. More buccolingual extension
D. Proximal B. More mesiodistal extension
C. More gingival
74. Partial Pulpotomy Technique is given by? D. More cervical
A. Bowen in 1974 (PGI June 2008)
B. Clark in 1976 83. The recommended concentration of fluoride in
C. Cvek in 1978 communal water supply is:
D. Willet in 1980 A. 0.1 ppm
B. 0.5 ppm
75. Cariogram is given by? (PGI June 2008) C. 1.0ppm
A. Aramani D. 2.0ppm
B. Blatterfein
C. Brathall 84. The spread of streptococcus mutans infection to
D. Klevin Allen the infant from a highly infected mother can be
prevented by:
76. Caries activity test measures? (PGI Dec 2008) A. Betadine mouthwash
A. Caries Susceptibility B. Oral hydrocortisone mouthwash
B. Motivational Aspect C. Oral topical chlorhexidine mouthwash
C. pH D. None of the above
D. None
85. Rampant caries is most commonly caused by:
77. After how many days following concussion A. Excessive sweet intake
should the tooth be checked again for pulpal B. Excessive bottle feeding
vitality? (AI 2007) C. Improper brushing technique
A. 24 hours D. Lack of calcimn intake
B. 10-12 days
C. 1 month 86. Pit and fissure sealants act by:
D. 3-6 months A. Killing microorganism causing caries
B. Blocking nutrient supply to bacteria
78. The major problem involved in performing C. Neutralising acids produced by bacteria
pulpectomy in primary teeth is: (AI 2007) D. All of the above
A. Root resorption
B. Inadequate operator skill 87. Chlorhexidine, a chemotherapeutic agent used
C. Limited knowledge of pulpal anatomy of for treating plaque, calculi and gingivitis is a:
deciduous teeth A. Bis-guanide
D. All of the above B. Quatinary ammonium compounds
C. Antibiotics
79. Best material for pulp capping is: D. Organic compound
A. Calcium hydroxide
Questions _ 457

88. A 7-year-old child consuming water supply 96. To achieve caries reduction diet modification
containing 3 ppm of fluoride will have mottling must be directed towards:
in: A. High calcium and vitamin intake
A. All teeth B. Foods that will buffer saliva
B. All permanent teeth, except third molars C. Detergent foods
C. Lateral incisors, canines, premolars, third D. Reduction of carbohydrate intake
molars
D. None of the above 97. The earliest evidence of bone fluorosis is evident
in a child consuming at least:
89. What is the recommended dose of fluoride for a A. I ppm
child from 0-2 years: B. 4ppm
A. Y4 mg C. 8 ppm
B. Y2 mg D. 16ppm
C. I mg
D. 2mg 98. Fluoride tablets containing sodium fluoride
90. Recommended daily dose of fluoride for a child contain:
A. 0.1-0.25 mg of fluoride
from 2-3 years:
A. Y4 mg B. 0.25-1.0 mg of fluoride
B. Y2 mg C. 1.0-1.25 mg of fluoride
C. I mg D. 1.25-1.5 mg of fluoride
D. 2mg
99. The most caries susceptible sites for caries sus-
91. Recommended daily dose of fluoride for a child ceptibility in children around 10 years of age
from 3-12 years: are:
A. Y4 mg .A. The mesial surface of central incisors
B. Y2 mg B. The occlusal surface of the first premolar
C. I mg C. The labial surface of the canine
D. 2mg D. The occlusal surface of the first molar

92. Which of the following procedures is replaced 100. The criteria for diagnosis of enamel fluorosis
by the use of occlusal sealants? (index) were given by:
A. Occlusal amalgam restoration A. Mckay
B. Topical fluoride restoration B. Dean
C. Prophylactic odontomy C. Black
D. All of the above D. Eager
93. For optimum results topical fluoride
101. Main site of absorption of ingested soluble
application should be administered:
fluoride takes place in:
A. Every three months
A. Stomach
B. Every six months
B. Small intestines
C. Every twelve months
C. Large intestines
D. Every eighteen months
D. None of the above
94. After topical fluoride application the instruc-
tions followed are: 102. The total amount of fluoride in the human body
A. Do not eat, rinse or drink for 30 minutes is about:
B.Rinse immediately but not eat or drink for 30 A. 2.3 gm
minutes B. 2.4 gm
C. Eat or drink immediately but do not rinse for C. 2.5 gm
30 minutes D. 2.gm
D. No specific instructions are required
103. Maximum amount of fluoride in the tooth
95. Which of the following sugars in human diet is structure is in:
considered the most caries promoting? A. Enamel
A. Sucrose B. Dentin
B. Fructose C. Cementum
C. Lactose D. Pulp
D. Glucose
458 • Dentogist: MCQs in Dentistry - Clinical Sciences

104. The concept of prophylactic odontomy was 112. The best pulp capping material is:
introduced by: A. Calcium hydroxide
A. Ripa B. Zinc oxide
B. Bowen C. Zinc phosphate
C. Hyatt D. Formaldehyde
D. Straffon
113. The best root canal sealer for a permanent
105. The pulp chamber in milk teeth in proportion tooth undergoing root canal treatment is:
to that of permanent teeth is: A. Zinc oxide engenol paste
A. Bigger in milk teeth B. Gutta-percha points
B. Smaller in milk teeth
C. Iodoform paste
C. Same in both teeth
D. Silver points
D. None of the above
114. Which of the following is not a medicament for
106. The best root canal sealer for a primary central
pulpotomy procedures in primary teeth?
incisor is:
A. Formacresol
A. Zinc oxide eugenol paste
B. Gutta-percha points B. Calcium hydroxide
C. Indoform paste C. Chlorhexidine
D. Silver points D. Glutaraldehyde

107. When there is a large exposure of the pulp in a 115. In the formacresol pulpotomy technique follow-
vital tooth with immature apex the treatment of ing the application of the pulpotomy medica-
choice is: ment which of the following is placed?
A. Apexifixation A. Cotton pledget
B. Apexogenesis B. Zinc oxide paste
C. Pulpotomy C. Calcium hydroxide paste
D. Pulpectomy D. Silver restoration

108. All the following are media to store an avulsed 116. Which ofthe following is indicated in a pulp
tooth except: exposure with coronal pulp hyperaemia:
A. Milk A. Formacresol pulpotomy
B. Saliva B. Pulpectomy
C. Oil C. Calcium hydroxide pulpotomy
D. Contact lens solution D. Extraction

109. The most common cause of failure of 117. Furcation involvement following necrosis of
pulpotomy using calcium hydroxide in a pulp of primary tooth is more common due to:
deciduous tooth is: A. Bigger size of pulp chamber
A. Internal resorption B. Faster spread of infection
B. Pulp calcification
C. Soft texture of the surrounding gum tissue
C. Tooth fracture
D. None of the above
D. Ankylosis
118. One of the effective and most commonly used
110. The periphery of the pulp surface in a primary
material for apexification is:
tooth is lined by:
A. Iodoform
A. Ameloblasts
B. Odontoblasts B. Calcium hydroxide
C. Unmyelinated nerve endings C. Zinc oxide eugenol
D. Loose alveolar type of connective tissue D. Zinc phosphate

111. Buckley's formocresol used for pulpotomy con- 119. The dental pulp first becomes inflamed by
sists mainly of: dental caries when the caries lesion reaches:
A. Formacresol A. Pulp
B. Distilled water B. Dentoenamel junction
C. Normal saline C. Half-way in dentine
D. Glycerine D. 1-2 rom away from pulp through dentine
Questions _ 459

120. The duration for application of calcium hydro- 127. Prolonged retention of a primary tooth may
xide in indirect pulp capping is: cause: (AI 2002)
A. 4-6 months A. Resorption of roots of adjacent teeth
B. 6-8 months B. Warping of alveolar bone in area
C. 4-6 weeks C. Ankylosis of succeeding permanent tooth buds
D. 6-8 weeks D. A change in path of eruption of succeeding
tooth
121. A mechanical pulp exposure of small diameter
(less than 1 mm in diameter) in a primary 128. The appliance used to treat thumbsucking:
mandibular first molar is treated by:
A. Crib appliance (AI 2002)
A. Direct pulp capping
B. Frankel appliance
B.. Indirect pulp capping
C. Forrnacresol pulpotomy C. Bionator
D. Calcium hydroxide pulpotomy D. Activator

122. Which of the following medicament can be used 129. What is 'the soft tisse profile of a child with a
as an antibacterial agent in therapeutic pulp malocclusion caused by persistent thumb
procedures? sucking: (AI 2002)
A. Calcium hydroxide A. Concave
B. Zinc oxide eugenol B. Convex
C. Formacresol C. Straight
D. All of the above D. Round
123. What is the thickness of calcium hydroxide
130. A dentist did restorative procedure in an 11-
layer in a tooth being treated by pulpotomy?
year-old child and found the primary molar in
A. 1 mm
various stages of exfoliation and there is slight
B. 1.5 mm
anterior crowding present between the anterior
C. 2.0mm
teeth. The dentist should give his next
D. 2.5mm
appointment? (AIIMS Nov 2007)
124. What is the most common cause of failure of a A. After three months for observation
pulpotomy that employs calcium hydroxide in B. After six months for recall examination
primary molars? C. After one year
A. Pulp fibrosis D. When all the permanent teeth will erupt
B. Pulp calcification
C. Ankylosis 131. Rate of space loss after extraction of
D. Internal resorption mandibular second molar is? (PGI Dec 2006)
A. 1 mm/year
B. 1.5 mm/year
C. 2.0 mm/year
125. In 8 year old boy permanent maxillary I" molar D. 2.5 mm/year
is ectopically erupting towards mesialy in an
intact arch with minimum root resorption of 2nd 132. In which of the following condition of
primary molar. Treatment of choice is: premature loss of primary teeth, the space loss
A. Disking of 10 molar (AI 2008) is maximum? (PGI Dec 2006)
B. Band and uprighting A. Maxillary molars, closing within 3 months
C. Wait and watch procedure B. Maxillary molars, closing within 6 months
D. Distalize with brass ligature wire C. Mandibular molars, closing within 3 months.
D. Mandibular molars, cosing within 6 months.
126. Which of the following dental sequele is likely
in child with a history of generalized growth 133. The early repair cleft is palate primarily done
failure (failure to thrive) in the first 6 months of for? (PGI June 2006)
life: (AI 2001)
A. Correction of speech
A. Retrusive maxilla
B. Correction of esthetics
B. Enamel hypoplasia
C. Retrusive mandible C. Induce the growth of maxilla
D. Dentinogenesis imperfecta D. Induce the growth of mandible
460 • Dentogist: MCQs in Dentistry - Clinical Sciences

134. Which of the following molar relationship is the 142. Late mesial shift ofthe permanent first molar is
most common in primary dentition? mainly a result of closure of:
A. Mesial step A. Primate spaces
B. Distal step B. Leeway space
C. Flush terminal plane C. Anterior spaces that failed to close during the
D. Can not say early mesial shift
D. Posterior extraction spaces
135. The type of space maintainer advised when 143. Primate spaces are related to the position of
there is bilateral loss of teeth on the same arch: spaces that are:
A. Distal shoe A. Between DC CD/CB BC
B. Band and Loop B. Between CB BCIDC CD
C. Band and Bar C. Between CB BCICB BC
D. Lingual arch holding appliance D. Between DC CDIDC CD

136. Oral screen: (AI 2005) 144. Mesiodens, which is a supernumerary tooth
A. Causes the child to breathe through nose leads to diastema between:
B. Allows for the passage of air through mouth A. I I
B. I I
C. Prevents passage of air through nose
C. Either A or B
D. Allows the passage of air through mouth and
D. Neither A nor B
nose
145. Removable orthodontic appliance causes tooth
137. The supervision "Of a child's development of movement by:
occlusion is most critical at ages: (AI 2006) A. Tipping
A. 3-6 years B. odily movement
B. 7-10 years C. Root movement
C. 11-14 years D. Torque
D. 14-17 years 146. Scars of orthodontics are:
A. Decalcification of teeth
138. With a flush terminal plane, permanent first B. Gingivitis
molars wiD: (AI 2006) C. Devitalisation of teeth
A. Initially be Class II D. All of the above
B. Initially be Class III
C. Immediately assume a normal relationship 147. The best age for giving a functional appliance
D. Erupt immediately into an end-to-end relation- for correcting skeletal dysplasia is:
ship A. 6-8 years in both boys and girls
B. 10-11 years in girls; 11-13 years in boys
139. Adenoid facies is seen in: C. After 16 years in both boys and girls
A. Mouth breathing habit
D. All of the above are correct
B. Bruxism 148. A functional appliance given In a patient with
C. Sub mandibular lymphadenitis class II molar and jaw relation, corrects the
D. Thumb sucking habit skeletal class II relation by:
A. Mainly stimulating mandibular growth and
140. Distal shoe guiding appliance is: partly by suppression of maxillary growth
A. Habit breaking appliance B. Maxillary inhibiting maxillary growth and
B. Space regainer . partially stimulating mandibular growth
C. Helps in eruption of first permanent molar C. Moving mandibular molars forward
D. It is a type of pulpectomy procedure D. Distallising maxillary molars

149. A headgear applied to maxillary molars in a


141. The early mesial shift of the permanent first patient with class II molar will correct the
molar is primarily a result of closure of: skeletal dysplasia by:
A. Primate spaces A. Down and backward rotation of maxilla
B. Leeway spaces B. Redirection of maxillary growth and posterior
C. Closure of extraction spaces positioning of maxillary teeth within the
D. All of the above maxilla
Questions _ 461

C. Redirection of mandibular growth 156. Mandibulofacial dysostosis is also called:


D. All of the above A. Achondroplasia
B. Treacher-Collins syndrome
150. The normal range of ANB angle is: C. Paget's disease
A. 2-4 0 D. Osteogenic imperfecta

B. 6-80 157. Form and function theory of growth was given


C. 8-100 by: .
D. 10-200 A. Roos
B. Moss
151. FMA angle denotes: C. Andreson
A. Frankfort-mandibular angle D. Black
B. Frankfort-mandibular plane angle
C. Frontal-maxillary angle 158. Dental age is determined from:
D. Frankfort-maxillary angle A. Which all teeth are present
B. Amount of root resorption of primary teeth
C. Amount of development of permanent teeth
152. Normal range ofFMA is:
D. All of the above
A. 30-40°
B. 25-30°
159. Teethusually emerge when:
C. 10-25°
D. 20-400
A. y., of their roots are complete
B. Yz of roots are complete
C. % of roots are complete
153. Premature loss of a non-restorable carious D. At full root formation
primary 2nd molar in the primary dentition can
have a severe adverse effect on arch perimeter 160. Which of the following teeth is not a succe-
in the transitional dentition. Treatment plan for daneous tooth?
this problem should be: A. Permanent maxillary canine
A. No treatment in the primary dentition a B. Permanent maxillary central incisor
removable acrylic space maintainer in the C. Mandibular I st and 2nd premolars
transitional dentition D. Maxillary and mandibular first molars
B. No treatment in primary dentition and removal
of the affected 2nd molar in the transitional . 161. The difference between the amount of space
dentition needed for the permanent incisors to erupt and
C. Distal shoe space maintainer in the transitional the splice available for them is called:
dentition A. Primate space
D. Distal shoe space maintainer in the primary B. Leeway space
dentition and a lingual or Nance arch space C. Interdental space
maintainer in the transitional dentition D. Incisor liability

162. An excessive curve of spee can be levelled by:


154. The major reason for replacement of prema- A. An absolute intrusion of the mandibular inci-
turely lost maxillary primary incisors, via a sors literally pushing them into the alveolar
fixed or removable appliance in the early process
primarily dentition is: B. Relative intrusion of the mandibular incisors,
A. Speech and aesthetics psychological reasons accomplished in a growing patient merely by
B. Form and function preventing their eruption while other teeth
C. Arch perimeter requirements in the transi- erupt normally
tional dentition C. Elongation of the canines and premolars while
D. Incising and mastication holding the incisors in their original position
D. All of the above
155. The terms 'Netrocclusion' Distocclusion and
mesiocclusion' in classifying malocclusion were 163. Correction of skeletal deep bite requires
given by: rotating the mandible:
A. Angle A. Downward
B. Lischer B. Forward
C. Bennett C. Upward
D. Profitt D. None of the above
462. Dentogist: MCQs in Dentistry - Clinical Sciences

164. During surgical exposure of impacted teeth,


flaps should be raised such that the teeth are
ulti-mately pulled through:
A Keratinised tissue 171. In a 9 year old child aU the permanent incisors
are present except the left maxillary central
B. Unkeratinised tissue
incisor. This may be due to: (AI 2001)
C. Alveolar mucosa
A Congenital absence
D. Either of the above
B. Insufficient root formation
C. Presence of a supernumerary tooth
165. Growth modification treatment in a case of D. Nasopalatine cyst
skeletal disproportions by means of a functional
appliance is aimed at:
172. The rubber dam is particularly adaptable to the
A Altering the expression of growth
primary 2 nd molar because the: (AI 2002)
B. Changing its direction A. Occluso-cervical height favours its retention
C. Changing the magnitude of growth B. Cervical constriction of crown favours its
D. All of the above retention
C. Mesiobuccal bulge favours its retention
166. Reverse head gear to pull the maxilla forwards D. Smaller diameter of crown favours its
in a skeletal class III case was designed by: retention
A Hostz
B. Delaire 173. Occlusal night guards are used to: (AI 2002)
C. Bimler A. Prevent bruxism
D. Frankel B. Reduce pocket formation
C. Redistribute forces on teeth
167. Bionator, which is a modified form of an acti- D. Permit eruption or elongation of teeth
vator, was designed by:
A Andreson 174. During what Stage of development is peer group
B. Robbin identity strongest: (AI 2003)
C. Harvold A Toddler
D. Balter B. Latency
C. Teenager
168. Herbst appliance was designed by: D. Prepuberty
A BirnIer
B. Pancherz 175. In a 5-year-old child tbe cause of aggressiveness
C. Stockli in dental clinic is? (AIIMS May 2007)
D. Hotz A. Fear
B. Separation
169. Herbst appliance, which is a functional app- C. Unknown
liance is a: D. Pain
A Removable appliance
B. Fixed appliance 176. The facial nerve degeneration occurs during
C. Removable from maxillary arch but fixed to normal forceps delivery used as?
the mandibular arch A Mastoid process is absent at birth
D. Removable from mandibular arch but fixed to B. Parotid gland is not fully formed
maxillary arch . C. The forceps hook only engages facial nerve
trunk (AIIMS Nov 2007)
170. The most probable age for a child to be in the D. Sublingual hematoma cause neuropraxia
ugly ducking stage is:
A 9 years 177. The method to measure the blood flow is?
B. 7 years A. Electric pulp test (AIIMS Nov 2007)
C. 11 years B. Percussion
D. 13 years C. Radiograph
D. Laser Doppler flowmetry
Questions _ 463

178. Untrue about Pit & Fissure Sealant is? 186. In which of the following condition the space
A. Depth of penetration odes not matter, full loss is most difficult to manage? (PGI Dec 2008)
length application is more important. A. A 5 yr old child with loss of a primary
B. Not used if tooth is cariess free for 4 years or mandibular 2nd molar
more. (PGI Dec 2006) B. A 91/2 yr old child with loss of a permanent
C. Not used in teeth with well coalesced pit & maxillary 1st molar.
. fissures. e. A 6 yr old child with loss of a primary
D. Indicated in teeth with deep pit & fissures maxillary 2nd molar.
with interproxirnallesion. D. A 7yr old child with loss of a primary
maxillary 2nd molar'
179. Dose ofMidazolam for conscious sedation by
Oral Route? (PGI Dec 2006)
A. 0.15 to 0.25 mg/kg 187. Which of the following is the emergency
B. 0.25 to 1.0 mg/kg treatment of choice' in case of traumatic injury
C. 1.0 to 2.0 mg/kg to tooth? (PGI Dec 2008)
D. 2.0 to 5.0 mg/kg A. Full veneer crown preparation with stainless
steel crown
180. Tooth start erupting during hich Nolla's Stage? B. A Stainless steel or preformed orthodontic
A. 4 (PGI Dec 2007) band around the tooth
B. 5 C. An Acid etched composite restoration
e. 6 D. A pin retained composite restoration
D. 7

181. In which formula to calculate Child Dose, 188. Fluoride in any form is contraindicated in:
weight is the parameter? (PGI Dec 2007) A. Chronic renal failure
A. Yiybg's Frmula B. Osteoporosis
B. Dilling's Formula C. Hypertension
C. Clark Rule D. Thyrotoxicosis
D. Cowling Formula
189. According to Standford-Binet, non-trainable
182. The acid used for Microabrasion Technique type of mental retardation has an IQ of:
recommended by CROLL is? (PGI June 2008)
A. Below 20
A. H2S0410%
B. Between 36-51
B. Hcl18%
C. Citric Acid 10% C. Above 51
D. None D. Between 90-130

183. Concentration of N20 used to produce 190. 'Canker sore' is otherwise commonly known as:
Dissociative Sedation and Analgesia? A. Recurrent apthous ulcer
A. 15 - 30% (PGI June 2008) B. Recurrent herpetic gingivitis
B. 20-45% C. Acute necrotizing ulcerative gingivitis
C. 50-70% D. Recurrent herpes labialis
D. >70%

184. According to Finn the best method of brushing 191. Rubber dam was introduced by:
in Mixed-Dentition and young Adult dentition A. Tomy Hanks
phase is? (PGI Dec 2008) B. SCBamum
A. Bass Sulcular Method C. Hedstorm
B. Horizontal Scrub Method D. GVBlack
C. Modified Stillman's Method
D. Fone's Method 192. The maximum dosage oflocal anesthetic agent
like lidocaine must be reduced when used in
185. Traumatic Injury to Primary tooth leads to
combination with a CNS and/respiratory
intrusion, after how much time the teeth usually
depres-sant because, it may result in:
re-erupts? (PGI Dec 2008)
A. Seizures
A. 30 Days
B. 3 Months B. Coma
C. 6 Months e. Death
D. 12 Months D. All of the above
464. Dentogist: MCQs in Dentistry - Clinical Sciences

193. Whkh of the following is not a type of cerebral C. Cervical third


palsy: D. Prognosis is equal in all cases, provided the
A. Spasticity tooth remains vital
B. Autism
C. Ataxia 199. An injury to the face of a 10-year-old boy resul-
D. Athetosis ted in class II fractures of the crowns of maxil-
lary central incisors and deep laceration of the
194. A child comes with fracture of maxillary central lower lip. Which of the following radiographs
incisor involving only enamel, but with suffi- are indicated for complete diagnosis of the
cient loss of tooth structure, such that it is of case?
concern to the parents and child aesthetically. 1. Radiographs of fractured teeth
The emergency treatment that should be given 2. Radiographs of mandibular incisors
is: 3. Bite wing radiographs
A. No treatment since the long-term vitality of 4. Soft tissue radiographs of the lip
the pulp cannot be predicted A. I and 2 only
B. Root canal treatment B. 1,2 and 4
C. Calcium hydroxide applied to the fracture site C. 2,3 and 4
followed by a stainless steel band or crown D. I and 4 only
D. Acid-etch composite restoration
200. The maxillary central incisors of a 4-year-old
195. An 8-year-old child who has sustained a child have become dark, yet the teeth appear
fracture of a maxillary permanent central normal. This discoloration was probably caused
incisor in which a large portion of the pulp is by:
exposed, presents for treatment three hours A. Tetracycline drug taken by the child for some
after the injury. What emergency treatment of illness
pulp should be performed? B. Exogeneous staining resulting from
A. Pulpotomy using calcium hydroxide vitamins/minerals taken by the child
B. Pulp capping using calcium hydroxide C. An undetected congenital hereditary factor
C. Pulpotomy using fonnacresol affecting calcification of the teeth
D. Pulpectomy and immediate root filling D. An unobserved or undetected traumatic injury
to the teeth with no resulting coronal fracture
196. An 8-year-old child with injury to maxillary
central incisor, with a large portion of pulp 201. An ll-year-old patient who has an Ellis class III
involved, was asymptomatic, had no acute pain, mesioangular fracture of a permanent
and did not present to the dental clinic for maxillary central incision presents to the dental .
several days. What treatment of pulp should be office 2 hours after injury. What emergency
performed at this time? . treatment should be preferred?
A. Pulpotomy using calcium hydroxide A. Pulpotomy
B. Pulpotomy using formacresol B. Partial pulpectomy
C. Pulpectomy immediate root filling C. Pulpectomy and immediate root filling
D. Pulpectomy and apexification D. Place calcium hydroxide and a zinc-oxide
eugenol dressing on the exposure site
197. A recently erupted permanent tooth has been
devitaUsed by trauma. Which medicament 202. A traumatised primary tooth turns pink over a
would you use while performing apexification period of time. The pathologic change that has
ofthe tooth? occurred is:
A. Zinc oxide eugenol A. Internal resorption
B. Calcium hydroxide, alone or with other B. External resorption
medicaments C. Ankylosis
C. Fonnacresol zinc oxide mixture D. Enamel hypoplasia
D. Corticosteroids
E. Amalgam 203. The green stains frequently seen as children's
teeth are caused by:
198. Which ofthe following areas ofroot fracture is A. Chromogenic bacteria
least conducive to a favourable prognosis? B. Material alba
A. Apical third C. Enamel defects
B. Middle third D. Dentin defects
Questions • 465
.
204. Which of the following caused bad breath? 208. A 4-year-old girl complains of a sore mouth.
I. Cleft lip She has painful cervical lymphadenitis and an
2. Draining fistula oral temperature of 38°C. Oral examination
3. Rhinitis reveals numerous yellow-grey lesions with red,
4. Cretinism margins on the palate, tongue and gingiva.
5. Recurrent herpes labialis Which of the following conditions is most
6. . Diabetes mellitus likely?
A. 1,3,4 A. Measles
B. 1,4,6 B. Erythema multiforme
C. 1,5,6 C. Herpetic gingivostomatitis
D. 2,3,6 D. Acute ulceromembranous stomatitis

205. Most common cause of acute generalised 209. A purulent lesion in .the facial vestibule of an 8-
gingival inflammation in a preschool child is: year-old is most likely:
A. Acute herpetic gingivostomatitis A. A pyogenic granuloma
B. Acute necrotising gingivitis B. An odontogenic fistula
C. Scurvy C. An infected apthous ulcer
D. Vitamin B deficiency D. An isolated herpetic lesion
210. A 3Yz-year-oldchild has an acute fever, diar-
206. A common cause of a draining fistula in rhoea, oral vesicular lesion and gingival tender-
gingival tissue in children is: ness. The most likely diagnosis is:
A. Periapical cysts A. Thrush
B. Drug allergy
B. Chronic periapical abscess
C. Apthous ulceration
C. Acute periodontal abscess
D. Acute herpetic stomatitis
D. Acute periapical abscess
211. In Dilantin hyperplasia:
207. A 14-year-old female has deep vertical pockets A. Oral manifestations are similar to those of
with bone loss of the mesial aspect of all first hereditary gingival fibromatosis
molars and some drifting of maxillary incisor. B. Follows a familial pattern
However, only minimal inflammation, a mini- C. Responds readily to antiinflammatory drugs
mal plaque are present. What is a probably D. More severe in adults than in children
diagnosis?
A. Gingivosis
B. Periodontosis
C. Periodontitis
D. Osteomyelitis
.............................................1.~.~.'!!~~?! ..~!.~~. ~~P~~.'!:.'!~~l!~!! .
II GENERAL II
1. B. (Difference in crystaIJine structure makes refractive index different)
Ref- Shobha Tandon 2nd ed. Pg 304

• Primary teeth are lighter in color- blush white .


• Also called milk teeth as their refractive index is the same as milk i.e. 1.
• Permanent teeth are darker in color- gray or yellowish white .

2. Ans. D. Partial trisomy 22


ReI Dorlands 28 th ed Pg -1626

Cats eye syndrome is associated with partial trisomy of 22 i.e. the presence of a patial additional copy of
chromosome 22

Cat'. Eye S~lIrome - pa~ 'JJ1somy,11 -x.

• Coloboma of iris (resembling vertical pupil of cat)


• Downslanting palpebral fissures
• Hypertelorism
• Anal atresia
• Congenital heart disease
• Renal malformations
• Skeletal abnormalities

3. A. (Constitutional delay in growth)


ReI - Ghai 6th ed Pg- 50, and CPDT 18'hed Pg- 938

• Short stature may result from normal variants of growth (Familial short stature and Constitutional Growth
Delay) or from Pathological Conditions.
• Physiological short stature resulting from normal variant of growth is more common than pathological short
stature .
• Amongst the option prov ided constitutional delay is the only example of physiological short stature and
hence the single option of choice.

4. A. (Inhaled short acting P2 agonist)


ReI Ghai 6'hed Pg - 358, 359
Inhaled short acting beta 2 agonists like salbutomal are the initial drugs of choice for management of
asthma exacerbation

Successful management of Asthma requires grading of severity of disease however the question does not give us
any information on the severity or grade of Asthma.

Amongst the options provided the drugs of choice to provide immediate symptomatic relief are Short
acting Beta - 2 agonists and hence the answer of choice.
Answers with Explanations _ 467

Management of Asthma
I I
Immediate Symptomatic Long Term Relief / Delayed
Symptomatic Relief
Short Acting beta 2 agonists Lon actin Beta 2 a onists
• Salbutarnol" - Salmetrol
• terbutaline' - Formoterol
• adrenaline • Theophylline / Aminophylline 1I'l
U
• Steroids .~

- Beclomethasone o
"0
* Inhalation rout is preferred - Budisonide o
"0
because of quick onset of - Fluticasone Q)
action and least side effects. c..
• Mast cell stabilizer;
- Sodium cromoglycates
- Nedocromyl sodium
- Ketotifen
• Leukotriene modifiers
- Monteleukast
- Zafarilukast
- Zileuton

5. C. (Trisomy 13)
ReI Ghai 6th ed pg- 593

• . Cleft lip, cleft palate, scalp defects suggest Trisomy 13 or Patau syndrome.
• Trisomy 13 and Trisomy 18 share several clinical features
• The highest discriminatory values are for ectodermal scalp defects and harelipand cleft palate in trisomy 13
and elongated skull and simple arches on all digits for trisomy 18

Trisomy: ..
• Most common abnormality in chromosome number (aneuploidy) is trisomy
• The most common trisomy is Down's syndrome (trisomy 21)
• A variety of other trisomies involving chromosome 8, 9, 13, 18 and 22 have been described
• Only trisomy 18 (Edwards syndrome) and trisomy 13 (Patau syndrome) are relatively common
• They share several karyotopic and clinical features with trisomy 21.
• As in Down's syndrome an association ith increased maternal age is also noted
• In contrast to trisomy 21, however, the malformations are much more severe and wide ranging As a result ,
only few fthese infants survive beyond the first year oflife most succumb within a few weeks to months .

Trisomy 13 (Patau syndrome):


• Cleft lip often midline
• Flexed-fingers with polydactyly
• Ocular hypotelorism
• Bulbous nose
• Low set malformed ears
• Small abnormal skull
• Cerebral malformation especially holoprosencephaly
• Microophthalmia
• Cardiac malformations
• Scalp defects
• Hypoplastic or absent ribs
• Visceral and genital anomalies
468 • Dentogist: MCQs in Dentistry - Clinical Sciences

Trisomy 18 (Edward syndrome)


• Low birth weight
• Closed fists with index finger overlapping the 3rd digit and the 5th digit overlapping the 4th
• Narrow hips with limited abduction
• Short sternum
• Rocket bottom feet

6. B. (Because of different angulation of the needle)


ReI Shobha tondon 1" ed pg- 469

Effect of mandibular foramen


Mandibular foramen is situated at a lower level than occlusal plane of primary teeth Therefore injection must be
made slightly lower and more posteriorly than for an adult patient.

7. Ans. A. Weight of the child


ReI KDT 6,h - ed Pg - 61, Sthed pg - 52- 53

I. Body Size - It influences the concentration of the drug attained at the site of action . The average adult dose
refers to individuals of medium built. For exceptional1y obese or lean individuals and for children dose may
be calculated on the body weight basis.

Individual Dose =; BW(kg) x Average Adult Dose


70

It has been argued that body surface area (BSA) provides a more accurate basis for dose calculation, because
total body water, extracel1ular fluid volume and metabolic activity are better paralleled by BSA.

Individual Dose =; BSA (m 2) x Average Adult Dose


70

The bod~ surface area of an individual can b~ calculated from Dubois Formula:
BSA (m ) BW (Kg) 0.425 X Height (Cm)O.72 x 0.007184
=;

Or obtained from chart-form or slide -rule nomogrms based on BW and height


However dose recommendations in terms of BSA are available only for anticancer and a handful of other
drugs, for the rest BW has been used as index.

2. Age- The dose of drug for children is often calculated from the adult dose :
Child Dose =; Age x Adult Dose (Young's Formula)

Child Dose =; Age x Adult Dose (Dil1 ing's Formula)

It can also be calculated (more accurately) on BW or BSA basis (see above) , and for many drugs,
manufactures give dosage recommendations on mg/kg basis Average figures for children are given below.

Aile Ideal BW (Kt!) BSA (m~) % or Adult Dose


Newborn 3.2 0.23 12.5
1 Month 4.0 0.26 15
3 Months 5.5 0.2 18
6 Months 7.5 0.4 22
1 Year 10 0.47 25
3 Years 14 0.62 33
5 Years 18 0.73 40
7 Years 23 0.88 50
12 Years 37 1.25 75
Answers with Explanations _ 469

8. ADS. A. Oral
Ref' Me Donald t h ed Pg- 308
1) INHALAnON
a. 85% dentists utilize thus route. This is most frequently utilized method.
b. Done with Nitrous Oxide (conscious sedation)

2) ORAL
a. By far the most universally accepted & easiest method
b. Particularly for pediatrics, no discomfort, pat ient readily accepts it
c. Iftaste is objectionable, drug may be mixed with a palatable liquid

3) INTRAMUSCULAR
a. Usually not preferred over oral route especially in younger age group
b. May be used if patient refuses to take medication orally or for some reason cannot do so.

4) SUBMUCOSAL
a. Deposition beneath the mucosa. It is almost exclu sively the province of dentistry.
b. Site usually chosen is buccal vestibule, particularly in area of maxillary primary molar or canine teeth
c. Term submucosal is often used interchangeably with subcutaneous

5) INTRAVENOUS
a. In hands of properly trained professionals, this can be the easiest, most efficient & safest next to
inhalation sedation.
b. Venipuncture is difficult to accomplish in very young of combatant child due to smaller vein size &
availability together with need to restrain the patient. Thus it is a consideration for apprehensive preteen
& adolescent patient.
c. This may be the route of choice in physically impaired & cogn itively disabled patient.

6) RECTAL
a. Very young children may do better with rectal than oral.
Rectal, submucosal & subcutaneous are rarely used.

9. ADs. (C) Submucosal


Ref' - Shobha london I'I ed Pg - 156

R OUll' \ of lndication s & hcn eflt s Limitutiun s ~I;.: r bk\


ad III i n isl ra 1ion
1. INHALATION • for mild to moderate anxiety • has weak potency
-Nittous oxide • analgesia for brief, comfortable • cannot be used in children with severe
procedures behavior problems
• rapid onset, early eliminations recovery • cannot be used in claustrophobic patients
.- • duration of action can be easily respiratory tract infections.
controlled
2. ORAL. • for pre-operative sedation • delayed onset & unpredictable
- several drugs • for all levels of anxiety absorption
used • better acceptability & ease • depends on patient compliance
• ARD less • difficulty in deterrni-ning drug dosage
a. Hyroxyzine (Vistaril) • mild sedative, anticholinergic, • better used in combination with other
(25 mg/5 00) antiemetic agents
• potentiates narcotics & eNS
depressants
b. Promethazine • Same as above • For mild anxiety only
(phenergan) • Better used in combination
(12.5 mg/See,
2.5 mg/5ec)
470 • Dentogist: MCQs in Dentistry - Clinical Sciences

c. Chloral hydrate • For all levels of anxiety • Not recommended in children below 6
[SOQmg{Scc] • Long working time yrs of age
• Believed to have a wide range of safety • No analgesia
• Maximum dose not to exceed 1500 m
3. intramuscular • Rapid onset of action • Injection required and will not be like by
(kctatnine, midazolam) • More reliable with little patient children
com liance • Ma cause in' durin administration
lj • Most rapid onset of action • Requires extensive armamentarium,
'.;:I • Permits titration and is easily reversible training
oc • Maintains a line for emergency drugs • Precautions to be taken in significant
"8 •
Best for invasive procedures of short hepatic or thyroid disease
~ duration
a.
10. Ans, C. Incapacitating communication and emotional problems
ReI Shobha london I" ed Pg- 554, Damle 2nd ed Pg - 423

AUTISM
• A severely incapacitating disturbance of mental & emotional development that causes problems in learning,
communication & relating to others.
• Manifests in first three years of life with no known cure
• Children have poor muscle tone (so spasticity is not seen because muscle tone is increased in spasticity)
• Poor coordination, hyperactive knee jerk, drooling, strabismus & epilepsy
• Have strict routines &prefer soft & sweetened foods thus increasing caries susceptibility
• Tend to pouch food instead of swallowing (due to poor tongue coordination)
• Exhibit extreme resistance to being held & show tantrums
• Language disturbance, parrot like repetitive speech difficulty in concept of yes & confusion in use of
personal pronouns
rd
• Mental retardation in Y2 to 2/3 children.

11. Ans. - A. Fluorosis


ReI Shobha Tondon ]'1 ed Pg - 542 - 543

CEREBRAL PALSY
~ A non-progressive lesion which occurs in developing brain before, during & after birth, leaving child with a
variety of neurological problems
~ 50% children die in infancy.
~ Motor deficit is fully evident only as the child develops
~ Etiology - decreased oxygenation to developing brain causing damage to brain .

~ Classification
• Based on anatomical Involvement
.:. Monoplegia - involvement of one limb only
.:. Hemiplegia - involvement of one side of body
.:. Paraplegia - involvement of both legs only
.:. Quadriplegia - involvement of all four limbs

• Based on neuromuscular involvement


.:. Spasticity
.:. Athetosis
.:. Ataxia
.:. Mixed
Answers with Explanations _ 471

Dental problems
• Increased caries rate than normal children
• Increased periodontal disease
• Children susceptible to trauma esp maxi.llary anterior teeth
• Excessive drooling & difficulty in swallowing
• Malocclusions occur twice often than average population (commonly protrusion of teeth , excessive
overjet & overbite, open bite & unilateral cross- bite)
• Persistent neonatal reflexes i.e. asymmetric neck reflex, tonic labrynthine reflex & startle refle x
• Mental retardation may be seen with seizures, speech disorders like dysarthria
• Spastic palsy patients present ith spastic tongue thrust, constricted mandibular & maxillary arches, class
II div II malocclusion usually with unilateral crossbite.
• Athetoid palsy presents with mouth breathing, tongue protruding between teeth & lips, bruxism, high
narrow palate & anterior open bite
• Ataxia palsy presents with staggering gait (lack of balance)

12. B. (Oxycephaly): Ghai essential ed pediatrics pgl602


I. Oycephaly
• Premature closure of coronal suture plus any other suture.
2. Brachycephaly
• Prematue closure of the coronal suture expands the skull parallel to the coronal
suture.
3. Trigonocephaly
• Premature fusion of metopic suture (part of frontal suture which joins the two halves
of the frontal bone of the skull.
4. Dolichocephaly or scaphocephaly -
• Premature closure of the sagittal suture, the skull grows perpendicular to the open
coronal sutures and appears to expand antero-posteriorly in the direction of the
sagittal suture.

13. A. (Carpenter syndrome): ReI Ghai Essent ial edition pediatrics 1602
Carpenter syndrome - it is genetically determineddisorder with autosomal recessive inheritance. In this
craniosynostosis is associated with mental retardation and preaxial polysyndactyly of the feet. Soft tissue
syndactyly of hands is also present. Patella is displaced. These children have short suture, are obese and often
suffer from congenital heart disease.
rd
14. A. (Negroes): ReI Bhalaji 3 l439
• Incidence of I in every 600-1000 births.
• The Negroid race has the least incidence (one in every 2000 births); while the mongoloids have the highest
incidence
• Unilateral clefts accounts for 80% of the incidence.
• Bilateral clefts accounts for 20% of the inc idence.
• Amont unilateral clefts, clefts involving the left side are seen in 70% of cases

15. D > A (Mineral content) > (Enamel prismatic arrangement): ReI Orban 's I t hl5?
16. B. (Oral rehydration therapy): ReI Ghai 5,h1249,250

17. C. (Canines, premolars and 2 Dd molars): ReI Wheelers 6'h124, Mcdonald 6'h1145,146
Chronology of the human dentition reveals that crowns of canines & 2nd premolars are completed by 7 yrs of age
& that of 2nd molar by 8 yrs of age & so are liable to tetacyclinic staining in a 6 yrs old child.
crowns of 1st molar is completed by 3 yrs and that of incisors by 5 yrs . Thus these teeth are not discolored in
a 6 yrs old child. '
Crown ofteeth are discolored ranging from yellow to brown & from greay to black.
Tetracycline will be deposited to a greater extent in dentin as compared to in enamel.
Tetracycline stains tends to fluoresce under UV light.
472 • Dentogist: MCQs in Dentistry - Clinical Sciences

18. C. (11 years): Ref NDB IJ-I April 1989 Q.54


Nothing much to say on this . It only signifies that one child is fast maturing & other is slow maturing. Now .
whether this change can be physiologic or pathologic, can be debated upon as somehow information in question
is incomplete. Whatsoever be the case it is a direct pick from NDB where answer is C. II yrs.

19. A. (12CDE6): Ref Wheelers 6Ih/24.


Tooth Flrit Evidence of Crown Erupti,on Root
~ . ClasslfkadoD .' completed .coaeted.
Central 3-4 mos. In utero 4 mos. 71/2 mos. 11/2-2 yrs
incisor
Lateral 41/4 mos. In utero 5 mos. 8mos. 11/2-2yrs.
Upper incisor
Jaw Canine 51/4 mos. In utero 9 mos. 16-20 mos 21/2-3 yrs.
First molar 5 mos. In utero 6 mos. 12-16 mos 2-21/2 yrs
Second 6 mos. in utero 10-12 mos 20-30 mos. 3yrs
Declduou molar
dentitIOn Central 41/2 mos in utero 4 mos. 61/2 mos 1 Y2 -2 yrs
incisor
Lateral 41/2 mos. In utero 41/4 mos. 7 mos. 11/2-2 yrs
I
tower incisor
Jaw Canine 5 mos. in utero 9 mos. 16-20 mos 21/2 -3 yrs
First molar 5 mos. In utero 6 mos. 12-16 mos. 2-21/2 yrs
Second 6 mos. In utero 10-12 mos. 20-30 mos. 3 yrs.
molar
Central 3-4 mos. 4-5 yrs, 7-8 yrs. 10 yrs
Permanent Upper incisor
dentition Jaw Lateral 10 mos. 4-5 yrs 8-9 yrs. 11 yrs.
incisor
Canine 4-5 mos. 6-7 yrs, ll-12yrs 13-15yrs.

20. D. (five in each quadrant): Ref Bhalajhi 2"d/49; Shobha Tandon l SI/54
I. Gum pads are divided into ten segments by transverse grooves .
2. Each of these segments consists of one developingdeciduous tooth sac
3. They are horse shoe shaped & develop in two parts : Labiobuccal portion & lingual portion. These are
separated from each other by a groove called dental groove.
4. Gingival groove separated the gum pad from the palate & floor of mouth
5. Lateral Sulcus: is the transverse groove between canine & I st deciduous molar segment. It is used in judging
the interarch relationship at a very early stage.

21. B. (Petitmal epilepsy): Ref Shobha Tandon 1"/544 & Harrison 16Ih/2358-2359

22. A. (Dentin is thicker in primary teeth): ReftShobha Tandon 2"d/304

23. A. (It is the age from the birth of the child): Ref Shobha Tandon 2"d/79

24. D. (Speech)
Tongue position: We are hardly aware of the tongue position in our day to day activities. (You'll notice it now,
while reading this line!). So definitely this is involuntary activity.
E.g. While masticating food, you don't have to think where to position your tongue? Respiration: I rather not
comment on this one!
Gagging: Take a spoon and gently rub against your posterior soft palate and you'll get the answer Speech: you
have to think what you have to speak; otherwise you may land up in trouble!
So the answer is Speech.
Answers with Explanations _ 473

25. A. (Subjective): ReI S. Tandon 1" 1132-133

"For most children, fear of dentistry is subjective and is not the result of previous dental care."
Subjective fears are those based on feelings and attitudes that have been suggested to the child by others
about him without the child's having had the experience personally .
Children have intense fear of unknown .
Objective fears are those produced by direct physical stimulation of the sense organs and are generally not of
parental origin.

26. A. (Mesiobuccal)

27. A. (Fear)
"The child's earliest fears associated with dentistry are those of the unknown and the unexpected ."
Now this is basically fear, option D says unknown i.e. the cause is not known .

28. A. (Addelston)
Other Modelling ~ Bandura
Home ~ Jordon

29. A. (Dr Evangeline Jordan)


Home = Handover mouth exercise ~ it's a behaviors modification method of aversive conditioning ~
introduced by Dr Evangeline Jordan in 1920.
Extreme Negative behavior ~ home technique physical restraint

Management is by
iWian
3-6 years old Child under 3 years of age
A child who can under- Physical, mentaland emotional
stand simple verbal handicap
compounds
A healthy child who can Handicapped child/immature
understand but who child straightened child.
exhibits defiance and
hysterical behaviour
during treatment.

Home Technique
After determining the child's behavior, the dentist firmly places his handover the child's mouth and
behavioral expectations are calmly explained close to the child's ear. When the child's verbal outburst is
completely stopped and the child indicates his willingness to cooperate , the dentist removes hand. Once the
child cooperates, he should be complimented for being quiet and praised for good behavior.
The whole procedure should not last for more than 20-30 second
Behavior management is defined as the means by which the dental health team effectively and efficiently
performs dental treatment and thereby instills a positive dental attitude.
Behavior shaping is the procedure which slowly deve-Iops behavior by reinforcing a successive approxi-
mation of the desired behavior becomes into being.
Behaviour modification is defined as the attempt to alter human behaviour and emotion in a beneficial way
and in accordance with the help of learning.
474 • Dentogist: MCQs in Dentistry - Clinical Sciences

CJassification of Behaviour Management


I. Pharmacological Methods
I . Pre-medication
a. Sedatives and hypnotics
b. Anti anxiety drug
c. Anti histamines
2. Conscious sedation
3. General anesthesia

II. Non - pharmacological Method


1. Communication
2. Behaviour shaping/Modification
a. Densensitization
b. Modeling
c. Contingency management (negative and positive reinforcement
3. Behaviour Management
a. Audioanalgesia
b. Biofeedback
c. Voice control
d. Hypnosis
e. Humour
f. Relaxation
g. Coping
h. Implosion therapy
I. Aversive conditioning -Home technique and physical restraint.

30. B. (Skinner)

Classic conditioning Pavlov


Operant conditioning Skinner
Psychoanalytic Sigmund Freud
Eight stage of life cycle Eric Erickson
Cognitive development Jean Piaget
Social cognitive learning theory Albert Bandura
Self psychology Harry sullivan and Heinz Kohut
Object relations Malanine Klien, Winkott is Fair Bairn

Operant Conditioning Theory


based on skinner's philosophy of radical behaviourism.
4 principles used
I. Positive reinforcement
2. Negative reinforcement
3. Extinction
4. Punishment

31. B. (Restoring carious teeth) ReI Shobha Tandon J"/544 ·


Arch length preservation can be best carried out by restoring canous teeth (either deciduous teeth or
(permanent class II restoration.
- The deciduous teeth are excellent natural space maintainers until the development/developing permanent
teeth are ready to erupt into the oral cavity.
- Space maintainers - Appliances used to maintain space or regain minor amount of space lost so as to guide
the unerupted tooth into a proper position in the arch.
Answers with Explanations _ 475

- Classification
- Removable i- Active/passives or functional/non functional
E.g.: Removable partial denture .
Fixed : BandedIBonded
Passive /Active
FunctionallNon functional/Cantilevered type
Functional: Pontic type
Non functional - Bar type, loop type, band type
Cantilever type - Distal shoe, band and loop type.

32. B. (Eruption is between 0-30 days): ReI Me donald 8th/Dentistry for the child & Adolescents /183
The neonatal teeth erupt any time within the first 30 days after birth. They do not have full tooth anatomy and do
not need to be extracted unless they cause feeding problems to the mother or the child.

33. D. (Initiation): ReI Me donald 8th/Dentistry for the child & Adolescents /52
Any disturbance during the stage of initiation and proliferation leads to congenital absence of one or more or all
teeth resulting in oligodontia or anodontia. Any disturbance in the other stages of tooth development after
initiation results in malformed teeth.

34. B. (Primate space):


These are referred to as primate spaces because they are also present in other primates besides man. It is more in
the mandible (1.7 mm) than in the maxilla (0.9 mm). They may be present in one arch and absent in the other.
Their absence usually results in crowding in the anterior teeth.

35. B. (Larger in deciduous teeth): ReI Me donald 8thIDentistry for the child & Adolescents /57
The pulp chambers in deciduous teeth are usually bigger relative to the size of the tooth and so the pulp chamber
and pulp horns are more superficial. This leads to faster exposure of the pulp and alveolar abscess formation in
deciduous teeth.

36. A. (Initiation): ReI Me donald 8th/Dentistry for the child & Adolescents / 129
A defect in initiation leads to oligodontia or anodontia whereas a defect in the later stages leads to malformation
of the tooth.

37. A. (0.9 mm in maxiUa and 1.7 mm in mandible): ReI Balajhi /44


The Leeway space is not fixed and may vary. It may be absent in some children. These figures are of unilateral
measurements.

38. A. (Mesial step relationship): ReI Balajhi/43


When this occurs it is most favourable for the permanent molars to go into a class I-relationship. If it is vice
versa it is referred to as a distal step relationship and when the distal surfaces are in one plane in the maxillary
and mandibular second molars it is referred to as a straight terminal plane relationship.

39. B. (Lateral incisor): ReI Shafer's Textbook oforal pathology 5th /64
The permanent lateral incisor is most commonly missing and its place may be sometimes filled with the
permanent cavine.

40. C. (Canine): ReI Balajhi /48


When the maxillary central incisors erupt they have a diastema due to their distal inclination . When the lateral
incisors erupt the diastema decreases and by the time the canines erupt the space usually closes completely.
476 • Dentogist: MCQs in Dentistry - Clinical Sciences

41. C. (Concrescence): ReI Shafer's Textbook oforal pathology 5th/55


Concrescence refers to this condition when the teeth are joined only at cementum. When the teeth are joined by
enamel and dentin they are labelled geminated when there are normal number of teeth with a single root canal. It
is termed fusion when there is joining of enamel and dentin with one tooth less and single root canal.

42. B. (Early mesial shift): ReI Balajhi /43


Early mesial shift occurs in a spaced dentition and late mesial shift of the Ist mandibular molar occurs in a
lJ nonspaced dentition and both bring the molars in a class I dental relationship.
';:1
r::
o
"C
43. D. (Eruption ofthe permanent incisors): ReI Contemporary orthodontics Proffit 4th/JJ9
o All the others bring about a decrease in the arch length whereas the eruption of the incisors specially the lateral
"C
tf incisors brings about an increase in arch length-referred to as secondary growth.

44. A. (An increase in arch length): ReI Contemporary orthodontics Proffit 4th/JJ9
The lateral incisor eruption brings about a lateral thurst thereby increasing the arch length .

45. D. (Self-correcting anomaly): ReI Contemporary orthodont ics Proffit 4th/449


This is one of the self-correcting anomalies and should not be corrected unless it persists after about 8 years of
age. The tongue has a forward thurst on these teeth which brings about this correction.
46. A. (First): ReI Balajhi /440
The process of organogenesis and morphogenesis usually occurs during the first trimester of pregnancy and
hence produces cleft lip and palate.
47. C. (Cleft palate): ReI Balajhi/84
The other three are syndromes or a collection of symptoms occurring with much less frequency as compared to'
cleft palate .
48. B. (Resorb more slowly than normal): ReI Contemporary orthodontics Proffit 4th/452
The eruption force of the permanent tooth is not the only force which produces resorption of the root of the
primary predecessor.

49. D. (Determining crown and rootshape): ReI Balajhi/38

50. C. (First and second premolars and second and third molar only): ReI Me donald 8th Dentistry for the child
& Adolescent
The tooth buds of all the other permanent teeth are initiated before birth.

CARIES AND PULP THERAPY

51. B. (proximal surface below contact area)


Ref Pediatric dentistry by Pinkham . 4th Ed Page 286

a. Primary first molars in both arches are less susceptible to occlusal caries because of their relative lack of deep
pits and fissures as compared to second molars.
b. The broad contact area between first and second primary molars contributes to a higher proportion of
proximal caries in these teeth . .
c. The caries susceptibility of the distal surface of the primary canine and the mesial surface of the primary first
molar is similar, and both are less.affected than first molar- second molar contact area.
d. In the mandibular arch, distal to the primary canine is free cleansing area. Hence, this is least susceptible to
caries .
e. Proximal caries usually starts gingival to the contact area and occlusal to the free margin of the tissue. (AIPG
09)
» Ifin the question permanent dentition is asked as to whether occlusal or proximal caries is prevalent. Then
the answer would be occlusal caries.
Answers with Explanations _ 477

For permanent dentition-


o Caries is most prevalent in faulty pits and fissures of the occlusal surfaces where the developmental lobes of
the posterior teeth failed to coalesce, partially or completely
o Primary occlusal grooves and fossae are smooth valley / saucer landmarks indicating the region of complete
coalescence of developmental lobes. Normally such grooves and fossae are not susceptible to caries because
. they are not niches for plaque and bacteria and frequently are cleaned by rubbing action of food during
mastication.
o Conversely occlusal fissures and pits are deep , tight crevices / holes in enamel where the lobes failed to
coalesce, partially or completely.
Ref-Art and science ofoperative dentistry, Sturdevant 5th edition/420 . Sturdevant, 5 th ed. Page 120.

52. Ans. A. Of furcation


ReI Shobha Tondon 1'1 ed Pg - 333

• In primary teeth, the earliest change is furcation involvement


• In permanent teeth, the earliest change is widening of periodontal ligaments.

Chronic Alveolar Abscess


Radiographically, the changes seen are in the form of a furcal radiolucency in primary molars, bone loss or root
resorption depending on the chronicity of the infection. -

53. B. (Oxygen saturation): ReI Shobha Tandon 1'1/332

54. A. (Cresol, formaldehyde, water and glycerin): ReI McDonald l h/425

55. A. (Iodoform, camphor, parachlorophenol and menthol): ReI Shobha Tandon 1'1/354

1h
56. C. (The primary teeth have a pronounced cervical constrtction): ReI Finn 4 /I50
"The greater constriction of the necks of primary teeth increases the danger of damaging soft tissue
interproximally as the gingival wall is established preparatory to shaping the proximal box. This endangers the
pulp if the wall is established too far gingivally. .

57. D. (Follow the direction of the enamel rods): ReI Sturdevant 4th/702
I. Ideally not more than 0.5 mm clearance with the adjacent proximal surface facially, lingually & gingivally
should be present.
2. Proximal ditch is cut approximately 2/3 at the expense of dentin and 1/3 at the expense of enmel. The 0.8
mm diameter bur end will cut approx . 0.5-0.6 mm into dentin and 0.2-0.3 mm into enamel.
3. Mesiofacial and mesiolingual walls is dictated by enamel rod direction & physical properties of amalgam.
Proper direction to proximal wall results in full-length enamel rods and 90° amalgam at preparations
margins.
4. Dentinal depth of the axial wall in proximal boxes of PM & molars should be same i.e. (i.e. 0.5-0.6mm)

58. D. (Will depend on radiographic findings): ReI see below


I. The root formation may not be absolutely complete which again would be recognized on x-ray. Here a
Ca(OH)2 pulpotomy here as at 10 yrs root of maxillary CI gets completed.
2. Hence a radiograph in most instances would show a mature apex of root.

59. A. (Mesiobuccal): ReI Finn 4Ih/I54,152


"Cl II cavities in all primary molars .....pulpal wall should be extended minimally in a buccolingual
dimension ecause ofthe possibility of nicking pulp hours. This is especially true in the mesiobuccal pulp horn
ara.
478 • Dentogist: MCQs in Dentistry - Clinical Sciences

60. C. (Pulpotomy and calcium hydroxide dressing): ReI Mcdonald 61h/437,516 Shobha Tandon 1st/338
Grossman 11th/10 2.
Pulpotomy is the surgical removal of the coronal pulp. Objective is the preservation of vitality of the radicular
pulp and relief of pain in patients with acute pulpalgia, Radicular pulp vitality is required for closure of root apex
so that apical seal can be formed before RCT.

61. A. (Lip biting): ReI Mcdonalds 6 Ih/305.


"Children who receive an Inferior Alveolar nerve injection for routine operative procedures may bite the lip,
tongue or inner surface of cheek resulting 24 hrs later in an ulceration often termed as "traumatic ulcer".

62. B. (To preserve the vitality ofrdicuIar pulp): ReI McDonald 's 6 Ih/437-441 ; Shobha Tandon 1"/342;
Grossman 1t h/ 102
I. Pulpotomy is the surgical removal of the coronal pulp.
2. Objective is the preservation of vitality of the radicular pulp and relief of pain in patients with acute pulpalgia
3. Radicular pulp vitality is required for closure of root apex so that apical seat can be formed befored RCT
Ponn~/t, .... ,u Jt"
Tissue fixation is evident Calcium bridge formation Better and non revsersible
Potent germicide Germicidal activity fixation of the tissues.
Vital tissue remains at the Vital pulp remains. Excellent antimicrobial
apex, clinical success: 65% Pulp tissue remains vital
Clinical success: 95% after 2 clinical success: 98-100%
years. Comparatively less dystrophic
Histological success 70% after Histological success: 35% calcification
3 years. Toxicity and Associated with internal Less pulpal necrosis
periapical leakage due to the resorption in deciduous teeth.
smaller molecular size. Bridging may make further
endodontic treatment
complicated

63. D. (Apexification): RefiMclsonalds 6,h/p 520-522; Grossman 11th/1 12, 113; Shobha Tandon 1/e p -357-360
1. "Apexification is a method to induce development of the root apex of an immature pulp less tooth
2. The aim of apexification is to induce either closure of an open apical third of root canal or formation of apica
calcific barrier against which obturation can be achieved
3. Is limited to coroal pulp only.
4. Most widely used & tested material is caldium hydrox ide (combination ofCa(OH)2 & CMCP can be used
5. Indication is nonvital permanent teeth with incompletely formed roots.

64. C. (Zinc Oxide Eugenol): ReI McDonald 's (/h/442; Shobha Tandon 1"/353 ,354
I. ZoE is probably the most commonly used filling material for primary teeth
2. Demerits of ZOE Paste
a. mild foreign body reaction on overfilling canal.
b. different rate of resorbtion of root & ZOE paste
3. Use of calcium hydroxide alone is not recommended in primary teeth as it can induce internal resorption in
primary teeth.

65. B. (Thickness of enamel and dentin): ReI Mcdonalds 61h/403


Differences between primary & permanent cavity preparation are because of following features in primary teeth.
a. thin enamel (1.0 mm)
b. broad molar proximal contacts
c. enlarged pulp chamber and high pulp horns
d. narrowed occlusal table.
e. enhanced cervical bulge .
f pronounced constriction at neck of tooth .
Answers with Explanations - 479

66. A. (Kri paste): ReI Mcdonalds 61h1442; Shobha Tandon 1'11353,354.


Root canal filling material for primary teeth, work "Kri paste" may be preferable , Excellent results have been
observed in many cases. Main advantagesof Kri paste are that Kri paste resorbs in synchrony with primary roots
& is less irritating to surrounding tissue, if a root is inadvertently overfilled"
Since iodoform paste doesnot set into hard mass. It can be removed if retreatment is required.

Composition of commonly used materials for priary teeth

- Parachlorophenol Iodoform 80.8% Zinc oxide - Calcium hydroxide


- camphor Camphor 4.86% Iodoform - Iodoform
- methol Parachlorphenol Thymol - Oily additives
2.025% Chorophenol
Menthal1.215% camphor
Lanolin

Since gutta pereha is not a resorbable material, its use I scontraindicated in the primary teeth.

67. A. (Zinc oxide eugenol): ReI Phillips l(jh1544

68. B. (0 2 saturation): ReI Shobha Tandon 1'11332

69. D. (Restoration of Class II, III cases): ReI Shobha Tandon 2ndl426

Mineral Trioxide Aggregate


• First described by Torabinejad for repair of lateral root perforations.
• It has been evaluated for several applications in dentistry such as root end fillings, direct pulp caps,
perforation repairs, apexification and pulpotomy.

Composition
• Tricalcium Silicate, Dicalcium Silicate, Tricalcium Aluminate, Calcium Sulphate Dihydrate, Bismuth Oxide.

Mechanism Of Action
• Stimulated cytokine release from bone cells .

nd
70. C. (Apexification in an immature traumatized pulpless teeth): ReI Shobha Tandon 2 l426

It is the first restorative material that consistently allow for the overgrowth of cementum , and it may fascilitate
the regeneration of the periodontal ligament.

71. C. (Has a pH of 7.0): Ref British Dental Journal 2000; 188: 427 -430

• Caridex contain butyric acid whereas Cariosolv contains leucine, lysine, glutamic acid. Both contain sodium
hypochlorite which is weak alkaline and the ultimate pH is alkaline in nature .
• In 1976 Goldman M. and Kronman J. reported on the effect of a N-monochloro-DL-2, arninobutyrate
(NMAB) solution used as a caries removal agent.

72. B. (Softening of dentin for removal of carious part):


Refhttp >llwww.dentistinla .comlstartseitelseitenlt2/cariosolv.htm
Cariosolv will soften and smooth the caries and then the caries can be wiped away without harming the dental
substance of your teeth and mostly painfree.
480 • Dentogist: MCQs in Dentistry - Clinical Sciences

73. B. (Lingual): Ref' Shobha Tandon 2nd/365

Tooth Preparation for Stainless Steel Crown


• Occlusal- 1.5-2.0 mm reduction
• Buccal & Lingual - 0.5 mm, least reduction lingually
• Promimal - 1-1 .5 mm reduction
• Gingival Termination - 0.5-1.0 mm into sulcus
V)
u • Gingival Finish Line- Feather Edge
'';:;
c:
0
"0
74. C. (Cvek in 1978): Ref' Shobha Tandon 2nd/407
0
"0
CLl 75. C. (BrathaII): Ref' Shobha Tandon 2nd/204
c,

76. A. (Caries Susceptibility): Ref' Damle 1s'/47

77. B. (10-12 days)


In trauma cases, the pulp vitality should be done in all cases and the teeth in the immediate area as well as
those in opposite arch should be tested.
But, the reliability of electric and thermal pulp testing methods immediately after trauma is debatable, as the
tooth is in a state of shock .
The failure of pulp to respond immediately after the trauma is not an indication for endodontic therapy.
Instead, vitality test should be repeated in a week to 10 days after emergency treatment.

78. C. (Limited knowledge of pulpal anatomy of deciduous teeth): Ref' s.G Damle 2nd/338
Limited knowledge of pulpal anatomy of deciduous teeth
The morphology of the root canals in primary teeth makes endodontic treatment difficult and impractical.
There are many variations in deciduous teeth, like lateral branching, connecting fibrils, apical ramifications
and partialfusion of teeth, which really make endodontic treatment in them difficult.

79. A. (Calcium hydroxide)


Calcium hydroxide remains the standard material for pulp capping in normal, vital pulp tissue.
The possibility of stimulating the repair process is good.
It is generally agreed that the prognosis of any pulp therapy technique improves in absence of micro-
organisms. * .
Pulp capping procedures should be limited to small exposures that have been accidentally produced by
trauma or during cavity preparation or true pinpointcarious exposures that are surrounded by sound dentin.
Calcium hydroxide is used because of its predic-tability in dentin bridge formation and maintenance of
vitality of the residual pulp. .

80. C. (Enamel, dentin and pulp)

Ellis Classification of Trauma to Anterior Teeth


Class 1: Simple #s of crown involving enamel
Class 11: Extensive #s of crown , with considerable amount of dentin involved but no pulp exposure.
Class Ill: Extensive # of crown, with considerable amountof dentin involved with pulp exposure.
Class IV : Traumatized tooth becomes nonvital (with or without loss of crown)
Class V: Tooth lost due to trauma
Class VI: # of root with or without loss of crown structure
Class V11: Displacement of the tooth without crown #
Class V11I: # of crown en masse
Class IX: # of deciduous teeth.
Answers with Explanations _ 481

81. B. (Pulp capping)

82. A. (More buccolingual extensions)


The gingival floor of the proximal box should be wide (that is more ofbuccolingual extension) so as to place the
margins in self cleansing areas . Due to the presence of board contact area.

Difference in cavity preparation in primary teeth arise due to


- Thin enamel covering (about I mm)
Enlarged pulp chambers
Narrow occlusal table
Broad molar proximal contacts
Enhanced cervical bilge
Marked constriction at neck.
Main difference in cavity preparation in primary teeth.

Class I Cavity
1. Isthmus not be more than 1/3rd the intercuspal distance in case of small carious lesion ~ due to narrow
occlusal table.
2. Depth should not be more than 0.5 mm into the dentin.
3. Pulpal floor- FLAT
4. Use of preventive resin restoration is advocated rather than the conventional cavity preparation which
includes all pits and tissues.

Class II Cavity
Due to presence of broad contact areas the gingival floor of the proximal box should be wide so as to place the
margin in self cleansing areas .
Box form should converse occlusally with the buccal and lingual wall paralleling the external tooth surface.
Walls of proximal box
Should meet the occlusal walls in the straight line to avoid any weak points and
Should not be flared as it would lead to unsupported enamel.
Isthmus should have just adequate width that it should not exceed 1/3rd the intercuspal width in primary
molars .
Axiopulpalline angle must be either rounded, tunneled on or grooved for sufficient bulk of the restoration.
Retention can be improved by a "U" shaped retention groove along the enamel dentinal junction of the
proximal box.
As the enamel rods, at the cervical area of the tooth are oriented occlusally the gingival seat should not be
beveled rather should follow the enamel rod inclination.

MESIOBUCCAL PULP HORN


Usually very high care should be taken to avoid damage while cavity preparation in molars.

83. C. (1.0 ppm): Ref Me donald 8th ed Dentistry for the child & Adolescent
The level of fluoride in the drinking water supply should be maintained at I ppm. Higher levels of 4 ppm and
above are desired in areas where deficient states are present.

84. C. (Oral topical chlorhexidine mouthwash): Ref Me donald 8th Dentistryfor the child & Adolescent /23 I
Chlorhexidine mouthwash is a strong antiseptic effective against Streptococcus mutans strain.

85. B. (Excessive bottle feeding): Ref Me donald 8th Dentistryfor the child & Adolescent /208
Rampant caries also referred to as nursing bottle caries or bottle baby syndrome is most consuming caused by
bottle feeding consuming sweetened milk specially at night.
482 • Dentogist: MCQs in Dentistry - Clinical Sciences

86. B. (Blocking nutrient supply to bacteria): ReI Me donald 8th Dentistry for the child & Adolescent 1356
Pit and fissure sealants block the nutrient supply to the bacteria thereby killing microorganisms and reducing
acid production.

87. A. (Bis-guanide): ReI Mc donald 8th Dentistry for the child & Adolescent 1248

88. B. (All permanent teeth, except third molars): ReI Me donald 8th Dentistry for the child & Adolescent I I 77
The third molars erupt at a very late age and are thereby not affected.

89. A. (Y4 mg): ReI Mc donald 8th Dentistryfor the child & Adolescent 1231

90. B. ReI Mc donald 8th Dentistry for the child & Adolescent 1231

91. C. (1 mg): ReI Me donald 8th Dentistry for the child & Adolescent 1231

92. C. (Prophylactic odontomy): ReI Soben peter essentials ofpreventive & community dentistry 2ndl380
Prophylactic odontomy given by Hyatt is a procedure is which the grooves and fissure are rounded off so that
food retention is reduced and thereby caries reduction.

93. B. (Every six months): ReI Me donald 8th Dentistry for the child & Adolescent 1229-230
This is the ideal time uptiII which the effect of topical fluoride application is effective.

94. A. (Do not eat, rinse or drink for 30 minutes): Ref Soben pet er essentials ofpreventive & community
dentistry 2nd1313
Eating , rinsing and drinking hampers the uptake of topically-applied fluoride .

95. A. (Sucrose): ReI Soben peter essentials ofpreventive & community dentistry 2ndl257
Sucrose is metabolised by inorganic bacteria to produce aids which dissolves enamel. It is consumed in
maximum amounts as compared to other sugars .

96. D. (Reduction of carbohydrate intake): ReI Soben pet er essentials ofpreventive & community dentistry
2ndl262
Carbohydrates once broken down by bacteria to produce acids which brings about caries.

97. C. (8 ppm): ReI Soben peter essentials ofpreventive & community dentistry 2nd/338

98. B. (0.25-1.0 mg of Ouoride): Ref Me donald 8th Dentistry for the child & Adolescent 1230

99. D. (The occlusal surface of the first molar): ReI Soben pet er essentials ofpreventive & community dentistry
2ndl244
These surfaces have pits and fissures which act as sites for food impaction on which cariogenic bacteria thrive to
produce acids . -,

100. B. (Dean): ReI Soben p eter essentials ofpreventive & community dentistry 2ndl340

101. A. (Stomach):
The absorption takes place of the soluble form of fluoride and occurs in the stomach and small intestines by
absorption and no active transport. .

102. D. (2. gm): ReI Soben peter essentials ofpreventive & community dentistry 2ndl286

103. C. (Cementum): ReI Soben peter essentials ofpreventive & community dentistry 2nd/293
Answers with Explanations - 483

104. C. (Hyatt) ReI Me donald 8th Dentistryfor the child & Adolescent/232

105. A. (Bigger in milk teeth):


The pulp chamber and also the pulp horns are bigger, more superficial and so more easily prone to exposure by
decay in a milk tooth.

106. A. (Zinc oxide eugenol paste): ReI Me donald 8th Dentistry for the child & Adolescent/40I

107. B. (Apexogenesis): ReI Me donald 8th Dentistryfor the child & Adolescent/470
A vital tooth with a wide open is treated by apexogenesis whereas a nonvital tooth is-treated by apexification by
incorporating various apex-inducing medicaments.

108. C. (Oil): ReI Me donald 8th Dentistry for the child & Adolescent/487
The rest are inert to the tooth and periodontal tissue and can be used to store an avulsed tooth for up to 6 hours or
more. The best results are obtained if the reimplantation is performed within 30 minutes.

109. A. (Internal resorption): ReI Me donald 8th Dentistry for the child & Adolescent/408
Calcium hydroxide induced odontoclasts formation which cause internal resorption the most common
complication of calcium hydroxide pulpotomy.

11 O. B. (Odontoblasts): ReI Me donald 8th Dentistry for the child & Adolescent/393
The periphery is lined by odontoblasts which are surrounding an inner layer of unmyelinated nerve endings and
blood vessels. The core of the pulp contains a dense plexus of these nerves and blood vessels.

111. D. (Glycerine): ReI Me donald 8th Dentistryfor the child & Adolescent/3I9
Buckley developed a type of formacresol which is a combination of one part of formacresol, three parts of
glycerine and one part of distilled water.

112. A. (Calcium hydroxide): ReI Me donald 8th Dentistry for the child & Adolescent/397

113. B. (Gutta-percha points): ReI Grossman endodontics practics 11th/242


Gutta percha is a thermoplastic material and is very inert. It brings about good vertical and lateral condensation
and hence its use.

114. C. (Chlorhexidine): ReI Me donald 8th Dentistryfor the child & Adolescent/399
Chlorhexidine cannot be used in contact with the pulp chamber as it brings about a severe anti-inflammatory
response. The other three-including 2% glutaraldehyde are good pulpotomy agents.

115. B. (Zinc oxide paste): ReI Me donald 8th Dentistry for the child & Adolescenti399-400
After the pulpotomy agent a thin mix of zinc oxide eugenol is placed up to a depth of2 mm and then followed by
a thick mix of the same paste.

116. C. (Calcium hydroxide pulpotomy): ReI Me donald 8th Dentistryfor the child & Adolescent/404

117. A. (Bigger size of pulp chamber): ReI Me donald 8th Dentistry for the child & Adolescent/57-58
As the bifurcation of the pulp is more superficial and the pulp chamber bigger, the chance of furcation
involvement are faster.

118. B. (Calcium hydroxide): ReI Me donald 8th Dentistryfor the child & Adolescent/404
Calcium hydroxide is a medicament which induces secondary dentin formation and hence is a very good agent
for pulp capping. It also induces apex formation therefore is a good apexification agent. .

119. B. (Dentoenamel junction): ReI Me donald 8th Dentistry for the child & Adolescent/309
As at the Dentoenamel junction the odonto-blasts start which transport the decay producing bacteria.
484 • Dentogist: MCQs in Dentistry - Clinical Sciences

120. D. (6-8 weeks) Ref: Me donald 8th Dentistry for the child & Adolescentl393

A. (Direct pulp capping): Ref: Me donald 8th Dentistry for the child & Adolescentl397
Direct pulp capping is performed during when the pulp is accidentally exposed to instrumentation and not when
exposure occurs during excavation of carious dentin.

D. (All of the above): Ref: Me donald 8th Dentistry for the child & Adolescentl404-405
Of all these medicaments formcresol is most effective but the others also exhibit this property.

C. (2.0 mm): Ref: Me donald 8th Dentistry for the child & Adolescentl404

D. (Internal resorption): Ref: Me donald 8th Dentistry for the child & Adolescentl408

ORTHODONTICS

125. A. (Disking of 1° molar)

126. B. (Enamel hypoplasia): Ref: Proffit 2nd164,65


I. Growth disturbances lasting 1-2 weeks or more, such as the one that accompanies birth or a growth cessation
caused by a febrile illness later, will leave a visible record in the enamel of teeth forming at the time (i.e.
Enamel hypoplasia
2. Premature infants can be expected to be sail throughout the first and into the 2 nd yrs of life
nd194,95
127. D. (A change in path of eruption of succeeding tooth): Ref: Bhalajhi 2
Prolonged retention of deciduous anteriors usually result in lingual or palatal eruption of their permanent
successors. Prolonged retention of buccal teeth results in eruption of permanent teeth either bucatty or lingually
or may remain impacted within the jaws.

128. A. (Crib appliance): Ref: Bhalajhi 2ndl106,108,338,; Proffit r d1126,127


"Mechanical aids like removable habit breakers incorporating cribs or hay rakes can be used"

129. B. (Convex): Ref: Proffit 2ndI126,127;Bhalajhi 2ndl106,108; McDonald's 6'hl776-780


"Excessive indulgence in thumb sucking habit can result in malocolusion characterized by flared & spaced
maxillary incisors, lingually positioned lower incisors, anterior open bite & narrow upper arch"
a. Labial tipping of maxillary anterior teeth resulting in proclination of maxillary anteriors.
b. Over jet increases due to proclined ncisors.
c. Lingual tipping of mandibular anteriors
d. Anterior open bite
e. Narrow maxillary arch.
f. Hypotonic upper lip & hyperactive mentalis activity

130. A. (After three months for observation)

131. A. (1 mm/year): Ref: Shobha Tandon 2ndl448


Rate And Time Of Space Closure-
• Seward (1965) found that a continual rate of closure 1.5 nun per year in the maxilla exised and all
individuals demonstrated space loss
• In the mandibule, the mean rate of spac closure was 1.0 nun per year, with individuals varying widely in the
timing of closure.
Answers with Explanations _ 485

132. B. (Maxillary molars, closing within 6 months): Ref Shobha Tandon 2nd/448,449

133. A. (Correction of speech): Ref McDonald t h/ 762


Palatoplasty - Closure of the palate is accomplished between 12 months and 2 years of age. The primary purpose
of palate closure by 2 years of age is to facilitate the acquisition of normal speech which correlates with the age
at which most children begins to talk. It may also improve hearing and swallowing by aligning the cleft palatal
musculature.

134. A. (Mesial step)

135. D. (Lingual arch holding appliance)


Lingual arch appliance helps in maintaining the space due to bilateral loss of dec. 2nd molar in mandibular arch.

136. A. (Causes the child to breath through nose)


Primary function of oral screen is to improve the tone and activity of perioral musculature.

137. B. (7-10 years)


A more better option would be 6-12 years
This age group is most critical to supervise occlusal development because it comprises a major part of mixed
dentition period
A developing malocclusion can be intercepted and prevented, better at this age group than any other of the
choices

138. D. (Erupt immediately into an end-to-end relation-ship)


Flush or Vertical Terminal Plane
Normal feature of the deciduous dentition
Its a relationship in which the distal surface of the upper and lower second deciduous molars are in one
vertical plane
On the same line, the erupting first permanent molars may also be in a flush or end on relationship
It is a favorable relationship to guide the permanent molars
74% of cases constitute flush terminal relation .
If the 2nd deciduous molar is in flush terminal plane, then the erupting permanent molar will also be in the same
relation. For this to change into class I relation the molar to move 2-3 mm in a forward direction (this is
accomplished by early and late mesial shift).

cted reliitiOn In a-nent dentitWlI


End- on or class I (due to early and late mesial step)
Class I or class III (due to further growth ofmandi-ble)
Class II

Flush terminal plane changes to class I molar relation by:


I. Early mesial shift - primate space utilized .
2. Late mesial shift - by using Leeway spaces.
Differential forward growth of mandible relative to maxilla

139. A. (Mouth breathing halut)


The type of malocclusion associated with mouth breathing is called long face syndrome/Adenoid facial
Long and narrow (Dollicocephalic and leptoprosopic) face, short and flacid upper lip, constricted upper arch and
anterior, marginal gingivitis or features

140. C. (Helps in eruption of first permanent molar)


It prevents the mesial migration of first permanent molar and so helps in eruption of first permanent molar
486 • Dentogist: MCQs in Dentistry - Clinical Sciences

141. A. (Primate spaces): ReI Balajhi 143


Primate spaces are present between the lateral incisor and canine in the maxilla and between the canines and the.
1st molar in the mandible in milk teeth. These spaces close when the 1st permanent molar erupts producing the
early mesial shift.

142. B. (Leeway space): ReI Balajhi 143


Leeway space is the space difference between the primary molars and the permanent premolars and is
VI
U
responsible for the late mesial shift of the permanent Ist molars.
.~
0 143. B. (Between CB BCIDC CD): ReI Balajhi 14 J
'0
0 These were first observed in all primates and hence the name. They may show individual variation.
'0
Q)
0..
144. A. (I 1): ReI Balajhi 1386
Mesiodens is the name given to an extra tooth between the upper permanent central incisors resulting in a
diastema and various types of malocclusions. Mostly it needs extraction.

145. A. (Tipping): ReI Balajhi 1272


The position of the wire of a removable plate and the direction of force are such so as to produce tipping
movement only.

146. D. (All of the above): ReI Contemporary orthodontics Proffit 4th1348-352


Apart from these root resorption is considered another scar. Fixed orthodontics in adult age is also considered a
social stigma.

147. B. (10-11 years in girls; 11-13 years in boys): Re/'salajhiI9-JO


This age takes the maximum advantage of growth and muscle force .

148. A. (Mainly stimulating mandibular growth and partly by suppression of maxillary growth):
ReI Balajhi1348-349
Most of the myofunctional appliances take this principle into account except for those which correct
dentoskeletal class III malocclusions.

149. B. (Redirection of maxillary growth and posterior positioning of maxillary teeth within the maxilla):
ReI Balajhil36 7
A head gear being an extraoral appliance applies force in such a direction so as to restrict maxillary teeth or it
can also retard growth of a fast growing chin.

150. A. (2-4 0 ) : ReI BalajhilJ 54

151. A and B. (Frankfort-mandibular angle & Frankfort-mandibular plane angle): ReI BalajhilJ 58

152. B. (25-30 0 ) : ReI Bal~jhilJ 58

153. D. (Distal shoe space maintainer in the primary dentition and a lingual or Nance arch space maintainer in
the transitional dentition): ReI Baiajhil223-224
The distal shoe space mantainer prevents further forward shift of the mandibular 1st molar and mantains the
space for the 2nd premolar and the Leeway space . The Nance lingual holding appliance prevents arch collapse
and so prevents reduction of space .

154. A. (Speech and aesthetics psychological reasons): ReI Bakljhil225

155. B. (Lischer): ReI Balajhil77

156. B. (Treacher-Collins syndrome): ReI Shafer's Textbook oforal pathology 5thl986


Answers with Explanations _ 487

157. B. (Moss): Ref Balajhi//7


Moss gave the form and function theory according to which the form 01 the teeth and thebones' IS aue to~e
function.

158. D. (All of the above): Ref Balajhi/213


These three factors help in determining the correct dental age and one alone cannot determine the same. This is
of great value in forensic evaluations.

159. C. (~ of roots are complete): Ref Wheler Dental Anatomy 8th/56


By this time the eruption force is sufficient to bring the teeth in the mouth and so also the tooth is able to bear
the forces which are given to it.

160. D. (Maxillary and mandibular first molars): Ref Wheler Dental Anatomy 8th/56
They do not have any predecessors and erupt distal to the last primary tooth.

161. D. (Incisor liability) Ref Balajhi /44

162. D. (All of the above) Ref Balajhi /56

163. A. (Downward): Ref Balajhi /437 .


In a deep bite condition the overbite is increased due to an upward rotation of the mandible which also reduces
lower facial height. A downward rotation of the mandible corrects this condition thereby opening the bite.

164. A. (Kerattnlsed tissue): Ref Balajhi /566


This is so because ultimately the exposed tooth should have normal texture and contour of attached and
unattached gingiva .

165. D. (All of the above): Ref Balajhi /329

166. B. (Delaire): Ref Balajhi /374

167. D. (Balter): Ref Balajhi /355

168. B. (Pancherz): Ref Balajhi /358

169. B. (Fixed appliance): Ref Balajhi /358


Contrary to the activator, Bionator and Frankel, the Herbst appliance is a type of fixed maxillary functional
appliance.

170. A. (9 years): Ref Balajhi /48


At this age the canines put pressure on the roots of the lateral incisors which in tum put pressure on the central
incisors thereby reducing their flaring resulting in a diastema - referred to as the ugly ducking stage.

II MISCELLANEOUS II

. 171. C. (Presence of a supernumerary tooth): Ref see below


I. Congenital absence of maxillary central incisor is rare
2. Mesiodens is most common supernumerary tooth.

172. B. (Cervical constriction of crown favours its retention): Ref Shobha Tandon }'1/284-287
"Primary teeth are sharpl constricted (bell shaped) cervically which certainly helps in retention of rubber dam in
primary teeth.
488 • Dentogist: MCQs in Dentistry - Clinical Sciences

Rubber Dam Isolation


- devised by S.c. Bannun in 1864.
- ensures appropriate dryness of teeth
- most successful method of isolating the operating field.

173. C. (Redistribute forces on teeth): ReI Glickman 8,h/556,557,I8I


I. "Maxillary stabilization appliance (occlusal nightguard) is the most universal and effective long term means
of interfering with the effects of bruxism.
2. The purpose of stabilization appliance is to redistribute the forces created by bruxium that alter mandibular
behaviour"
3. It does not prevent bruxism rather decreases the ill effects of bruxism

174. C. (Teenager): ReI Kaplans Psychiatry 7,h/ Vol JI/2862,255I


Although peer group is important in prepubertal age group but its maximum importance is in adolescence where
major physical, psychological, cognitive & social transformation occurs.

175. A. (Fear): ReI Finn 4,h/17

176. A. (Mastoid process is absent at birth): ReI Shobha Tandon 1"/47

177. D. (Laser Doppler flowmetry): ReI Shobha Tandon 1"/332

178. D. (Indicated in teeth with deep pit & fissures with interproximal lesion): ReI Shobha Tandon 2 nd/260

th/3I8
179. B. (0.25 to 1.0 mg/kg): ReI McDonald 7

0.25 to I mg/kg to a maximum single dose of20 mg


0.1 to 0.15 mg/kg to a maximum dose of 10 mg
Slow I V titration. See manufactur's recommended dosage guidelines.
Syrup - 2 mg/ml
Injectable - Img/ml and 5 mg/rnl vials

180. C. (6): ReI Shobha Tandon 2nd/91.93


I. Absence of crypt
2. Initial Calcification
3. 1/3 of the crown completed
4. 2/3 of the crown completed
5. Crown almost completed
6. Crown completed
7. 1/3 of the root completed
8. 2/3 of the root completed
9. Root almost completed
10. Apical end of the root completed

"-, 181. C. (Clark Rule): ReI Nelson pediatrics lO,h/288

Clark's Ruld = Child Weight in Pound . x Adult Dose


150

182. B. (HclI8%): ReI McDonald 7,h/I22


Answers with Explanations _ 489

183. B. (20 - 45%): Ref Shobha Tandon r'n«!


Plane II: Dissociation Sedation and Analgesia-Concentration of 25-45%
• Psychological symptoms such as dissociation or detachment are felt. A euphoric state, similar to
• alcoholic intoxication (the laughing gas parties).
• The patient is suffused by a warm wave and may experience a slight humming or buzzing in the ear,
. • With a floating feeling.
• Reduction in blink rate.
• Patient is conscious and responds to questions, with however, a considerable mental effort involved in ~
• thinking ~
o
'0
o
184. D. (Fone's Method): Ref Finn 4,hl540 '0
Q)
Q.
Fones Technique. In this method, with the teeth in occlusion, the buccal and labial surfaces are brushed with a
large circular motion . The lingual and occlusal surfaces are brushed with an in-and-out horizontal brushing
action. .

185. C. (6 Months): Ref Shobha Tandon 2ndl59i

186. .A. (A 5 yr old child with loss of a primary mandibular 2nd molar): Ref Bhalahji 3rdl226

187. C. (An Acid etched composite restoration): Ref Shobha Tandon 2nd1588,589

188. A. (Chronic renal failure)


Because fluoride is mainly excreted through kidneys .

189. A. (Below 20)

190. A. (Recurrent apthous ulcer)


Conker sex also called as recorrect apthous.

191. B. (SC Barnum)

192. D. (All of the above)

193. B. (Autism)
Types of cerebral palsy
Spasticity
Athetosis
Atoxia
Rigidity

194. D. (Acid-etch composite restoration): Ref Me donald 8th Dentistry for the child & AdolescentI46i-462-463
Although pulpal prognosis cannot be deter-mined at the time of injury, there is no contraindication to placing an
aesthetic restoration at the emergency visit. Acid-etch composite restoration is best, since no burring of tooth
structure is needed any further damage to a recently concussed pulp is avoided .

195. A. (Pulpotomy using calcium hydroxide): Ref Me donald 8th Dentistry for the child & Adolescentl466
If a child comes within 72 hours after injury, pulpotomy with calcium hydroxide is indicated . A tooth with a
wide immature apex is considered a good candidate for this technique because of the recuperative young pulp.
490 • Dentogist: MCQs in Dentistry - Clinical Sciences

196. D. (Pulpectomy and apexification): ReI Me donald 8th Dentistry for the child & Adolescentl466-467
Pulpectomy is indicated if the exposure is of longer than 72 hours duration since the pulp is generally infected
beyond recovery. Apexification will allow apical constriction to occur in a nonv ital immature permanent tooth
and subsequently enable complete closure of root apex by conventional endodontic procedures.

197. B. (Calcium hydroxide, alone or with other medicaments): ReI Me donald 8th Dentistry for the child &
Adolescentl468-469
VI
U 198. C. (Cervical third): ReI Me donald 8th Dentistry f or the child & Adolesc entl493
-.:;
C This is because of difficulty of stabil ising the crown segment and because of easy access of oral microorganisms
o
"C to the fracture area .
o
"C
Q.)
0.. 199. B. (1, 2 and 4): ReI Me donald 8th Dentistry for the child & AdolescentI459-460-461

200. D. (An unobserved or undetected traumatic injury to the teeth with no resulting coronal fracture) ReI Me
donald 8th Dentistry for the child & Adolescentl459
The injury caused rupture of intrapulpal blood vessels which eventually leads to darkening of the tooth structure.
The tooth may first appear yellow and goes undetected and becomes noticeably dark only after a period of time.

201. D. (Place calcium hydroxide and a zinc-oxide eugenol dressing on the exposure site): ReI Me donald 8th
Dentistry f or the child & Adolescentl465
Since the tooth is vital and has a small mechanical pulp exposure, it will respond very favourably by a direct
pulp capping procedure using calcium hydroxide and zinc oxide eugenol.

202. A. (Internal resorption): ReI Me donald 8th Dentistry for the child & Adolesc entl471

203. A. (Chromogenic bacteria): ReI Me donald 8th Dentistry for the child & Adolescentl447-448
Chromogenic bacteria cause stains to form in the material alba on the teeth .

204. D. (2, 3, 6):


Patients with diabetes mellitus produce ketones that produce sweet breath orders . Patients with draining fistula
or rhinitis have disagreeable breath odours from putrescence.

205. A. (Acute herpetic gingivostomatitis): ReI Me donald 8th Dentistry for the child & Adolesc entI417-419
Acute herpetic gingivostomatitis usually occurs in preschool children and develops suddenly, with general
malaise, light fever, acute gingivitis and small vesicles in oral mucosa.

206. B. (Chronic periapical abscess): ReI Me donald 8th Dentistry for the child & Adolescentl390
Draining fistulas are chronic conditions caused by long-standing pressure on the bone and soft tissue that
produces a tract from the site of infection to the oral cavity.

207. B. (Periodontosis): ReI Me donald 8th Dentistry for the child & Adolescentl436-438
Periodontosis, also called juvenile periodontitis tends to occur in relatively clear mouth, more frequently
between the ages of l4~29 and more often in females than males .
208. D. (Acute ulceromembranous stomatitis): ReI Me donald 8th Dentistry for the child & Adolescent1422-423

209. B. (An odontogenic fistula): ReI Me donald 8th Dentistry for the child & Adolescentl390
An 8-year-old has many unerupted teeth, which leads frequently to an odontogenic cyst, which gives rise to an
odontogenic fistula.

210. D. (Acute herpetic stomatitis): Ref' Me donald 8th Dentistry for the child & AdolescentI417-419

211. A. (Oral manifestations are similar to those of hereditary gingival fibromatosis):' ReI Me donald 8th
Dentistry for the child & Adolescentl426
Dilantin sodium is a drug given for epileptic seizures. The most common complication of this drug is gingival
hyperplasia, which resembles hereditary gingival fibromatosis.
HIGH YIELD FACTS
• American Society of Dentistry for children advocate a routine dental appointment on or before I st birthday.
• 1945 1st artific ial water flouridation-Grand Rapids- Michigan
• 1.955 Acid etch technique described - Buonocore
• 1964 CREST- I st ADA - approved flouridated tooth paste
• Expressed consent - can be written/oral
• Implied consent - determined by the behaviour of Patients
• Retrocuspid papilla - lie attached gingival of mandibular canine.

Common Abnormalities seen in children

• Fissured tongue - autosomal dom inant


Common in Down's Syndrome, Mouth Breathers
• Leukoedema - increased in blacks , increased with age
• White sponge nevus - Autosomal Dominant
• Thrush - acute pseudomembranous candidiasis.
• Geographic tongue - females
Erythema migrans ~ lesions on buccal and labial mucosa and palate
• Aphthous ulcer - common in female
a. :s
Minor form - 1em size resolves in 1-10 days solitary
b. Major form-Sutton's disease multiple, crateriform ~ 1 em resolve in 3-6
weeks
c. Herpetiform - uncommon
,l.
Involves Nonkeratinised Mucosa

• Secondary herpetic ulcer


,l.
heals in 7-14 days
,l.
Site: Vermilion of lips, hard palate, attached gingival, perioral region
Treatment: Acyclovir

• Erythema Multiforme
Male, severe variant is Stevens-Johnson syndrome

• Median Rhomboid glossitis


Males, Diamond shaped , anterior to circum variate papillae,
• Squamous papilloma caused due to papilloma virus
• Verruca vulgaris/common wart-HPV
• Condyloma accurnnatum/venereal wart - HPV
• Papillary hyperplasia - hard palate - cobble stone appearance
• Pyogenic granuloma - gingiva, lips, tongue, Buccal mucosa
• Soft tissue abscess/Parulis ~ gingival absess, gum boil
Gingival and alveolar Mucosa
• Peripheral ossifying fibroma
Gingiva, a anterior to Ist permanent molars involves interdental papilla, arises from PeriosteumlPDL
• Peripheral giant cell granuloma (PGCG) - females- gingival anterior to Ist Molar
• Hemangioma - Ist year of life - females
Rx: Excision, laser ablation, sclerosing agents , steroids, cryotherapy
492 • Dentogist: MCQs in Dentistry - Clinical Sciences

• Lymphangioma
J-
Ist Decade
Neck lesion ~ cystic hygroma
Rx: Excision

• Neurofibrom atosis
autosomal dominant Rx Cosmetic
CI'l
U
'';:; • Gingival granular cell tumour of newborn/congenital epulis, in the anterior Gingiva (HIP) Arbikosov cells
C
o
"'0 PLEOMORPHIC ADENOMA
o
"'0
Q)
c.. Most common site
a. Intraoral ~ Hard and soft palate
b. Extraoral ~ Parotid
c. Most common benign sublingual gland neoplasm
d. 2nd mucoepidermoid carcinoma
• Mucocele ~ lower labial mucosa , buccal mucosa, ventral tongue
• Ranula - floor of mouth, sublingual gland
• COC/Godin cyst - Incisor to canine region of maxilla and mandible associated with unerupted tooth
• AOT common in females-unerupted tooth, anterior maxilla
• Compound odontoma -anterior maxilla
• Complex odontoma - females - posterior mandible
• Unicystic ameloblastoma is most common variant in children. Least aggressive
• Cherubism-males-bilateral-autosomal dominant- premature exfoliation of primary teeth. Rx cosmetic recontouring
• Gardner's syndrome ~ multiple osteomas
• Fibrous dysplasia - unilateral, maxilla associated with Albright syndrome
• Traumatic bone cyst - scalloping between roots of teeth
• Langerhans cell disease / Histiocytosis 'x' floating teeth
• Histiocytosis 'y' ~ verruciform xanthoma
• Burkitt's lymphoma - starry sky appearance
J-'
Males, floating teeth. Posterior mandible-associated with EBV
• Ewing's sarcoma - posterior mandible, femur, pelvis (onion skin/sun burst appearance)
• Osteosarcoma - males - symmetric widening ofPDL
• Sun burst appearance, maxilla and mandible post region femur, tibia
• Brachycephaly - seen in Down syndrome
Cephalic Index: Dolichocephaly - < 75
Mesocephaly 75-80
Brachycephaly > 80

• Sturge-Weber syndrome - angiomatosis and calcification ofleptomeninges, sei~res, portwine nevi of face
• Ha~lermann-Streiff syndrome - dyscephaly, mandible hypoplasia hypotrichosis
• Rieger's syndrome-iris dysplasia, midface hypoplasia protruding umbilicus
• Seckel syndrome-dwarfism, microcephaly; facial hypoplasia, low set lobeless ears.
• Hemifacial microsomia due to hematoma of stapedial artery
• Syndrome with hyper and hypodontia
1. Crouzon's, 2. Down's, 3. Oro-facial digital, 4. Hallermann-Streiff
High Yield Facts _ 493

• Fusion - primary teeth have two separate pulp chambers


,!.
Bifid crown with one chamber
• Gemination - single pulp chamber familial
• Dens evaginatus - evagination of inner enamel epithelium cells
• Dens ivaginatus - invagination of inner enamel epithelium - tooth Withina tooth Maxillary lateral incisors
,!.
Resembles Supernumerary Tooth on X-ray
• Taurodont - failure of proper level of horizontal invagination of Hertwig's epithelial rootsheath
• Congenital: ichthyosis
,!.
a. Hyperkeratosis of knees and elbows
b. Fish like scaly skin
c. Delayed tooth eruption
d. Root dilaceration
• Amelogenesis Imperfecta : Types
Type I autosomal dominant
Type II hypomaturation
Type II D Snow capped teeth
Type III hypocalcified
Type IV hypomaturation - hypoplastic with taurodontism

COMMON VIRAL INFECTIONS IN CHILDREN

I. Herpes simplex virus


,!.
a. Vesicles on mucosa oflips, tongue, gingiva acute phase generally lasts 7-10 days
b. Steroids and antibiotics contraindicated .i .
c. Newly introduced topical antiviral agent - Penciclovir
d. Recurrent herpes simplex ~ reactivation of virus lying dormant in trigeminal ganglia

2. Herpes zoster / chickenpox ~ Shingles / Zona


Occur in dorsal spinal or cranial nerve ganglion during winter and spring
3. Herpangina
,!.
Under 4 years
Coxsackie A virus 2,3,4,5,6,8, 10
,!.
a. Disease in epidemics
b. Summer, fall
c. Self limiting
4. Hand, foot and mouth disease
,!.
Coxsackie A-16 virus
,!.
a. Epidemics, vesicles incubation period 2-6 days
b. Children between 1-10 years age
c. Self limiting
494 • Dentogist: MCQs in Dentistry - Clinical Sciences

COMMON BACTERIAL INFECTIONS IN CHILDRENS


I. Impetigo
,J,
Streptococci or staphylococci
Bullous ~ bullae/clean eroded lesions - Staph. aureus
Nonbullous ~ crusted lesions ~ causes glomerulonephritis, history of milk trauma, aSYJDRtomatic or pruritis
Rx: Nonbullous - washed with antiseptics
Topical/systemic antibiotics
Bullous type: Rx Penicillinase resistant Pn ~ Dicloxacillin
2. Scarlet fever
,J,
Caused by ~ hemolytic streptococci
Erythrogenic toxin
Sore throat, fever, vomiting
Bright red papular skin rash beginning in skin folds is spreading to remainder of body.
Strawberry tongue and Raspberry tongue
Lymphadenopathy
,J,
Rx: drug of choice Penicillin

Candidiasis
• Pseudomembranous ~ curdy plaques
KOH preparation of smear reveals hyphae
Rx: I Mild ~ topical antifungal
II Moderate ~ systemic antifungal - ketoconazole
III Severe ~ sy.stemic antifungal - amphotericin B intravenous, highly. nephrotoxic

1. Diabetes Mellitus Type I


a. Children genetically predisposed
b. Islet cell antibodies
Periodontal disease , increased bone resorption, inflammatory gingival changes,
Xerostomia
Recurrent intraoral abscesses
Enamel hypoplasia and hypocalcification.
Both advanced - delayed eruption of permanent dentition.
Rx: Intermediate acting insulin iliPH, Lente)

2. Acute Lymphoblastic Leukemia


Most common childhood malignancy ~ Acronym: SMALL, cervical and submandibular lymphadenopathy

3. Sickle cell anemia


Occurs around 6 months age,
painful swelling of dorsum of hand/foot due to iscbaemic necrosis of metacarpal or metatarsals,
hepatosplenomegaly, .
pallor, cardiomegaly, icterus, asplenic,
stepladder appearance, hair on end (also in Thalassemia)
effect of skull enamel hypomineralization, increased periodontal disease.
Rx: Bone marrow and cord blood transplantation
High Yield Facts _ 495

4. Hemophilia A (Classic Type)

Persistent bleeding involving oral cavity


Prolonged (PIT)
Factor VIII assay
I 2-5% moderate
II 5-30% mild
III < 2% severe deficiency III
o
Rx: a. Fresh frozen plasma ',0::
c
b. Cryoprecipitate o
"0
c. Lyophilized factor 8 precipitate o
"C
Q,)
Q.
EACA inhibits fibrinolysis by blocking activation of plasminogen to plasmin
DDAVP-l-deamino-8-D- arginine vasopressin
Proplex or konync - activated factors that bye pass factor 8, massive doses of factor, and plasmapheresis
High risk procedures
1. Block anesthesia
2. Simple extraction, curettage, gingivoplasty
3. Multiple extraction, flap surgery, gingivectomy, apicoectomy
4. Orthognathic surgery
5. Endotracheal intubation

5. AIDS
Estimated time from birth to onset of severe symptoms in HIV infection children 6.6. years
Mean survival time - 9.4 years
Oral candidiasis is the most common opportunistic infection in children with HIV
Hairly leukoplakia Rx: oral acyclovir
Kaposi's sarcoma Rx: Palliative excision, chemotherapy, radiation therapy.

CHILD PHYSIOLOGY

RESPIRATORY SYSTEM

• Childs chest wall is more elastic than that of adult


• Sternum is less rigid
• Adult number of alveoli present at 6th year of life
• Children have a greater proportion of alveolar surface area to lung size
• Functional Residual Capacity (FRC) ~ volume of gas remaining in lungs at the end of a normal expiration is less
than that of adult.
• Pediatric Alveolar Ventilation (AV) /Functional Residual Capacity (FRC) is almost 5 times of an adult ~
anesthetized with lower gas concentration
• Heart rate in newborns = 120
• Adult heart rate reached by 10-12 years of age
• Infant heart is relatively inelastic
• Mean systolic BP in newborns = 75-85 mm Hg
496 • Dentogist: MCQs in Dentistry - Clinical Sciences

GI SYSTEM

• Infants -4 secrete low levels of acid


• Adult levels not reached until 3 years of age
• Longer gastric emptying time
• Low levels of cytochrome P-450 }
• Decreased glucoronyl transferase Enzymes
• Decreased pseudocholinesterase

GFR

• Adult levels 5 times the newborn level


• Adult GFR is reached at I year age
• Total body water equals 80% of infant's weight 50-60% of normal adult body weight
• Decreased serum albumin and plasma globulin
Infants adults
Total body H20 80% 50-60%
Extracellular 35-40% 20%
Intracellular fluid 40-45% 40%
• Fat makes up 10-15% of full term newborn weight
• BZDs and barbiturates bind to fat. Care while administering these drugs
• Child's height triples, but weight increases 20 fold
• BSA is high in neonates, adults = 1I6thof neonates weight.
• Nomogram -4 chart with height, weight , BSA example: 'west, Talbot
• Cognition - awareness of internal and external environmental influences on oneself
• Visual analogue score - measure degree of discomfort in children line 100 mm long anchored on either side by
happy-sad faces ,
• Voice control -4 for unco-operative or inattentive but communicative child
• HOM ~ defiant, obstreperous/hysterical child
• Nitrous oxide - 02. inhalation
.J..
Contraindications
Chronic obstructive pulmonary disease
I. Severe emotional disturbances
2. Ist trimester of pregnancy
3. Drug/disease induced pulmonary fibrosis
• Three most feared or anxiety producing stimuli
i. Injection of LA
ii. Application of rubber dam .
iii. Tooth preparation with high speed handpiece

LOCAL AN ESTHESIA

• Gate control theory 1965 - Melzack and Wall


• Transcutaneous electrical nerve stimulation (TENS)-Woolf 1989
Activation of large diameter, primary afferent fibre, intum inhibit small fiber transmission
Met-enkephalin - l st Opioid peptides isolated
Enkephalins -4 endogenous opioids

• Three opiate receptors -4 mu, kappa, lambda


• LA block Na channels
• Depress rate of depolarization
• Pain isoneoflstmodalitiestobeblocked followed by cold, warmth, touch
• Benzoic acid ester derivatives
Benzocaine, Procaine (Novocaine), Tetracaine (Pontocaine), Chloroprocaine (Nesacaine)
Allergic reactions
• Amides
Lidocaine 1943 ~ amide derivatives of diethyl aminoacetic acid

..
Mepivacaine (carbocaine), prilocaine (citonest)
Bupivacaine (marcaine), etidocaine (duranest) en
U
"
• Potency ~ is the concentration required to achieve desired effect of nerve blockade e
(Procaine has lowest intrinsic potency); (lidocaine, prilocaine, mepivacaine ~ intermediate); (Tetra, bupi, o
"C
etidocaine- high potency) o
"C
Q)
• Procaine, chloroprocaine~ for spinal anaesthesia Q.
• Lidocaine I% with Adrenalin (I :2,00,000) duration, 416 min with infiltration duration of anaesthesia
• Children
Vasoconstrictor concentration I: I ,00,000 adrenaline
• Bupivacaine, etidocaine - most cardiotoxic
Maximum Recommended Dose Absolute Max
Lidocaine 4.4.mglkg 300mg
Prilocaine 6 400mg

ANALGESICS USED IN CHILDREN

• Aspirin 10-15 mglkgldose given at 4 intervals maximal limit 3.6 glday


• Acetaminophen - Tylenol, Tempra, Datril most common analgesic in pediatrics in US
Disadvantages fatal liver necrosis ~ 3 g for children, 15 g for adults
• Narcotic analgesics act in CNS
• Opiates ~ act at periaqueductal gray matter

CONSCIOUS SEDAnON
Definition: Minimally depressed level of consciousness In which the patients ability to maintain a patent airway
independentlyand continuously and to respond appropriately to physical stimulation or verbal command.
• GA is a controlled state of unconsciousness accompanied by a loss of protective reflexes, including the ability to
maintain an airway independently and respond purposefully to physical stimulation or verbal command
• Nitrous oxide
Nitrous Oxide - 02
Sedative concentration 30-45%
Maximal concentration usually should not exceed 50%~ Indicated for mild to moderately anxious ~ Potentially
cooperative child.
Most commonly encountered side effect ~ nausea
Nitrous Oxide analgesia is relatively safe and effective for the Rx of children in dental office .
Tingling sensation in fingers and toes ~ verifies sedation.
Upon termination of Nitrous Oxide administration, inhalation of 100% O2 for not less than 3-5 minutes
recommended.

I. Intramuscular route of administration might be considered easiest of all routes.


Mid deltoid region, vastus lateralis of thigh, gluteus maximus ~ 3 regions where usually administered
(intramuscular).
2. SC route
J.,
Rate of absorption is slower due to sparse blood supply
3. Intravenous ~ optimal and ideal route for conscious sedation most difficult
Thrombophlebitis ~ rare complication
498 • Dentogist: MCQs in Dentistry - Clinical Sciences

Sedative Drugs

• Sedative hypnotics ~ act on Reticular Activating System


Barbiturates
Non-barbiturates, chloral hydrate, paraldehyde
• Antianxiety agents ~ limbic system ~ seat of emotions ~ anger, fear, rage
• Narcotics - opioid receptors ~ periaqueductal gray matter
• Oral chloral hydrate ~ most common sedative for children, bitter taste
• Minor tranquilizers ~ antianxiety agents
• Major tranquilizers ~ anti psychotics
• Ketamine ~ dissociative agent
Act on thalamus, cortex

Monitors During Sedation and GA

1. Pulse oximeter - degree of 02 saturation of haemoglobin in patients blood and heart rate
3. BP cuffs
4. Precordial stethoscopes
5. ECG
6. Temperature probes
7. Defibrillators
• Pulse oxime ter (most useful and accurate for detecting developing hypoxia)
• Capnograph - to determine whether the Patient is breathing, analyses C02 content of respired gases
During GA, record vital signs at a minimum of5 min interval

Mode of Action of Various Antibiotics

• Inhibit cell wall ~ penicillins, cephalosporins, monobactams, carbapenems, glycopeptides, Azole anti fungals
• Inhibit protein synthesis.
Ribosomes
50s ~ macrolides, chloramphenicol, lincosamides
30s ~ aminoglycosides, Tetracyclines
• Alter cell membrane permeability:
Polymyxin, clotrimazole, polyene antivirals
• Inhibit nucleic acid synthesis:
Rifampin, griseofulvin, nucleoside antivirals
• Inhibit cYJocbrome sterol ~ Azoles

Epinephrine

• Drug of choice for life threatening reactions and basic cardiac life support, severe asthmatic reactions
• Sympathomimetic - stimulates a and ~ receptors' has antihistaminic action
• im/sc
• Side effects ~ HTN (Hypertension), cardiac arrhythmias, anxiety, headache

Diphenhydramine ~ HI Antagonist

• Side effects - Sedation, anticholinergic


High Yield Facts _ 499

Diazepam (Valium)
• Used to Rx status epilepticus
• Side effects - Sedation, respiratory depression

Hydrocortisone Na Succinate
• Treat acute adrenal insufficiency
• Severe anaphylactic reactions or asthmatic attack
Glucagon raises serum glucose levels by encouraging glycogenolysis
Ephedrine : used to raise BP and heart rate from shock levels
Release catecholamines
• Side effects - hypertension, tachycardia, arrhythmias, headache
• Atropine used to treat bradycardia - parasympatholytic
• Side effects - tachycardia, arrhythmia, dry mouth
• Lidocaine - to treat ventricular arrhythmias
• Cardiac lidocaine- I mg/kg iv

Airway
Head tilt, chin lift or jaw thrust maneuver.
Heimlich maneuver - operator stands behind the patient, places one hand below xiphoid process over mid upper
abdomen clenches it with other hand and pulls up and back forcefully.
Pressure on baroreceptors of carotid sinuses may precipitate reflex bradycardia.

AUergic Reactions

Epinephrine at 0.0 I mg/kg administered.


Diphenhydramine - oral - at 6-intervals for 1-2 days after any allergic reactions 02'
Steroids
Hyperventilation
Decreased PaC0 2 - hypocarbia
Rx:02
Asthma - IgE mediated

Diabetes Mellitus (DM)

Oral Glucose - if patient is conscious


Sugar dissolved in water
50% dextrose iv -+ unconscious
glucagon im if iv not possible
- Breaks down glycogen and increases blood glucose

3 Major Core Functions of Public Health


1. Assessment
2. Policy development
[3. Assurance
• [EPSDT - Early and periodic screening, diagnostic and treatment. Identifies problems of children as early as
possible provides comprehensive, preventive and remedial care] .
• Project Head Start -+ preschool program serving children from low income families. 1965
• Baby clinic (Bebe-clinica) 1986 - Brazil
• Macri - sling/chair like device provides comfortable position with minimal restraint - invented by professors at
Baby clinic
• Baby clinic -+ uses 3 step process
1. Educational Programme
2. Preventive Programme
3. Restorative Programme
500 • Dentogist: MCQs in Dentistry - Clinical Sciences

Definitions

Neonate - 1-4 weeks postpartum (i.e, I month)


Infant: I year oflife
Toddler 1-3 years
Child 3-12 years
Adolescent 12-19 y,ears

Growth and Development


1st division of ovum into 2 cells usually takes place in 24-36 hrs
Period of ovum - 1-14 days
Period of embryo 2-8 weeks - organogenesis
Period of fetus - 8-40 weeks
4-8 months ~ babbling, 4 distinct syllables
9-12 months ~ symbolic gestures
12-18 months ~ 10 words
18 months-2 years ~ 2 word sentences

• During I st year of life, most children undergo 50% increase in length and 200% increase in weight.
• By the end of2 years, child's brain is 75% of adult weight
• Upper lip formed by fusion of median nasal , lateral nasal and maxillary process
• 2 palatal shelves fuse at 8 weeks, i.e. 2 months in Utero
• Muscles of mastication derived from I st Branchial Arch,.
• Muscles facial expression 2nd Branchial Arch
• Symphysis closed by 2nd year
• Child stands alone at II months / 9-16 months
• Walks alone at 11.7 months / 9-17 months

Stages of Life Cycle of Tooth - By Orban 1957

• Growth
• Initiation ~ 6 weeks - bud stage
• Proliferation - tooth genn - cap stage
• Histodifferentiation - bell stage
• Morphodifferentiation - dictates final size and shape of tooth - advanced bell stage
• Apposition - along DEI and DCJN ~ Dentinocemental junction
• Calcification- begins at cusp tips and incisal edges
• Eruption
• Attrition
• Korff's fibres ~ assist in structural support to develop dentin
• Hertwig's epithelial root smeath - responsible for size , shape of root and eruption of tooth.

Eruption Phases

1. Pre-eruptive phase - tooth forms to movement towards oral cavity


2. Prefunctional eruptive phase - tooth root is app. 1/2 -2/3 the final length
3. Functional phase ~ tooth meets its antagonist

Face

• Face begins at 4 weeks IUL


• Primary teeth begin to form at 7 weeks in utero
• Enamel of all primary teeth is complete by I st year of age
• All primary teeth are erupted by 24-36 month ofage
• Root structure of primary teeth complete by 3 years
High Yield Facts _ 501

• At birth ~ 24 teeth calcified ~


• Ist primary tooth to erupt ~ A lA, Ist permanent tooth 616
• Primate spaces ~
leD
• 1st permanent molar is the 1st tooth to show germ formation at age 3 liz -4 months in utero
CI, LI ~ 5 - 5 liz months
Canine 5 liz - 6 months
4,5,7, 8 ~ after birth VI
o
.~

Mussen 1984 - 4 Major Areas of Cognitive Develop During 1st Year Of Child's Life c:::
o
• Area of perception
-g
• Recognition of information i
c..
• Categorise
• Enhancement of memory
• conditioning classic ~ when 2 stimuli are paired together
operant/instrumental ~ occurs when child's actions are reinforced/rewarded
• Cognitive development - Jean Piaget 1952
• 1st 2 years ~ sensorimotor develop ~ 6 stages
• Mean explosive vocabulary of 18 months old ~ 10 words, 3 years ~ -1000 words
• Fear of strangers ~ 7-12 months age
• Fear of Separation from Parents
Starts at 6 months
Peaks between 13-18 months
• Non-reflexive smiling: 2-3 months
• Temper tantrums: 1-2 years old, terrible between 2-3rd year
• Bottle caries affect maxillary incisors mandibular incisors spared
• Caries is the disease process
• A child's 1st visit to Dentist should occur no later than.12 months age

Examination of a Patient

Parent and dentist sit facing each other in a knee-to-knee position, supporting the child with the head cradled on dentists
lap

Anticipatory guidance format

• Birth to 6 months - clean teeth daily anticipatory guidance for with soft brush during bath and after feeding
• 8 years - ugly duckling stage
• 10 years - sealants for 2nd molars
• 12 years - child becomes responsible for own oral health

Definitions

• preterm births - gestational age under 37 weeks


• low birth weight babies - infants weighing < 2500g
• very low birth weight infant weighing < 1500 g
• extremely low birth weight babies infant weighing < 1000 g

Water Fluoridation
• Most effective means of reducing caries
• Most cost-effective, most convenient, most reliable
• 40-50% caries reduction in primary dentition
• 50-65% caries reduction in permanent dentition
502 • Dentogist: MCQs in Dentistry - Clinical Sciences

• liquid supplements without vitamins ~ 0.5 mg/ml


• liquid supplements with vitamins ~ 0.25 mg/ml and 0.5 mg/rnl
• F tab - 0.25,0.5, l rng F
• F vit combination 0.5, 1 mg doses
• Omno fluoricheck water analysis kit
• Hand held colorimeter- Hach DR 100 colorimeter
Hachco Loveland, CO
• Halo effect ~ fluoridated beverages in non-fluoridated areas
• I mg ofF = 2.2 mg NaF

Concentration of Fluoride on water

Age <0.3ppm 0.3-0.6 ppm >0.6 ppm


Birth-6 month 0 0 0
6 months-3 years 0.25 mg 0.25 mg 0
6 years-l 6 years 1 mg 0.5 mg 0

• Placenta is not an effective barrier to passage of Fluoride to fetus


• Most sensitive age for inducing fluorosis in maxillary incisors of a typical child is 22-26 months at outer extremes
18-30 months age
• Lethal dose ofF for a typical 3 years old child is approximate 500 mg
• No more than 120 mg of supplemental F be prescribed at anyone. time
• If excess fluoride ingested
Vomiting induced by 15 m1 Ipecac syrup with 6 oz of water for children
Administer milk
• Flouride varnish was Ist introduced in Europe in 1964 Trade name - Duraphat caries reduction 25-75%
• Off-label use - Therapeutic use of F varnish for caries prevention in US (Off label - usage of a thing for which it is
not intended to be)
• FD & C ~ Federal food, drug and cosmetic Act
• Duraflor and Duraphat are 5% NAF (2.26% F ion)
• Varnish application preceded by professional prophylaxis
• Varnish causes teeth to appear yellowish
• In high risk persons, F varnish applied at 3-6 months
• Human milk causes a greater fall in plaque Ph than bovine milk
• Infant should be held by the parent while feeding from the bottle
• Infants total nutritional needs be supplied from milk or formula until 5-6 months of age
• Bacteria for dental disease are present at the eruption of primary teeth
• Once teeth have erupted, a wet, soft bristled brush is gently wiped over the teeth
• Primary dentition - Flossing not required until interdental contacts established.
• Fetus starts sucking and swallowing at 3-4 months in utero
• Non-nutritive sucking ' Rooting reflex - disappears at 7 months
Sucking reflex - intact till 12 months
• Transitional swallow ~ common at 3-10 years age
• Full adult swallow 3-4 years observed, usually present by 9-10 years
• Pacifier never attached to a ribbon or string around child's neck
• Pacifier should be sturdy - I piece - nontoxic, flexible, firm have easily grasped handles
• Inseparable nipples and mouth guards have a label warning against tying it around neck
• Most injuries to primary teeth occur at 112- 212 years (Toddler stage)
• Cardinal signs of abuse ~ injuries in various stages of healing , tears of Labial frenum, repeated injuries
• Primary dentition frequently displaced/luxated than fractured because of pliable alveolar bone
• Concussion ~ tooth is tender to biting and percussion
• Mobility - tooth is loosened but not displaced from socket
High Yield Facts _ 503

• Avulsion - completely displaced from alveolus


• Children acquire active immunity to Tetanus by 4-6 years
• Booster at 11-12 years
• Every 10 years thereafter
Periapical radiolucency seen on X-ray, 3 weeks after injury
Replacement resorption evident at 6-7 weeks after injury
, Plan postoperative radiographs at I months and 2 months after injury
Pulp's initial response to trauma is pulpitis
Pulpal hemorrhage - discolouration occurs within 1-2 days after injury. Necrotic pulp discolouration weeks-months
• 90% of calcified primary teeth resorbs normally
Rx: Not indicated
• Inflammatory resorption of primary teeth
Rx: Resorbable Zinc oxide paste - RCT
• Exposed dentin in tooth fracture is covered with acid resistant Ca(OH)z paste or with OIC then restored with acid
and composite resin technique
• 3/4ths of root formation be present to consider pulpectomy
• Canals instrumented 1-2 mm short of apex
• Fractures is in middle/cervical3rd of root indicate extraction
• Concussion Rx: tooth taken out of occlusion
• Intrusion 2-6 months taken to re-erupt
• Primary teeth avulsed need not be re-implanted
• X chromosome resembles a middle-sized autosomal chromosome in size
• Y chromosome resembles one of the smaller and acrocentric autochromosome
• Short arm of chromosome ~ p. (p=petit)
• Long arrrr-e q
• FISH - Flurescence in situ hybridization
DNA Analysis
• Recessive disease ~ cystic fibrosis, sickle cell disease
• Polygenic/multifactorial inheritance ~ Hypertension, cancer, adult on set-diabetes, allergies
• Methylated gene is turned off - imprinting ,,,
• Neurofibromatosis von Recklinghausen disease
Dominant inheritance
Birth marks - cafe-au-lait spots, more than neurofibroma I

Neurofibromatosis

Soft signs ~ short stature relative macrocephaly 87%


Croves sign
Unexplained Hypertension' severe headaches, benign glial tumors of brain , sarcomas and schwannomas
Severe bleeding during surgery ~ tumour highly vascular pheochromocytoma
17q11 .2 - Plexiform neurofibromas

Tuberous Sclerosis

Dominant
Facial angiofibromas
Hypomelanotic macules ~ Ash leafspots seen using UV light (Wood Lamps) ,
Thickened patches of waxy appearance skin - shagreen patches.
Subungual fibromas causing nail deformities, seizures, tubers of brain, rhabdomyomas of heart cysts of kidney and
angimyolipomas, renal dysplasias, dental ~ Enamel pits on facial surface of incisors and canines.
Fibrous nodules on gingival, tongue, palate.
504 • Dentogist: MCQs in Dentistry - Clinical Sciences

Marfan Syndrome
• Autosomal dominant CIT disorder long and narrow face with high palatal vault , prominent lower jaw, laxity.of
joints, lengthening of long bones - sternal deformity, scoliosis, dislocation of lens, myopia .
• Altered CIT of aorta
• Fibrillin gene on chromosome 15 - responsible

Ehlers-Danlos Syndrome - 10 Types


lj
.~

c Hyperlaxity ofj oints, increased stretchability, velvety softness of skin, easy bruising, poor wound healing with scarring.
o Spontaneous vase/bowel rupture leading to death.
"0
o • Type IV - vase and bowel rupture - deficiency in release of type 3 collagen
"0
,f • Type VIII - premature loss of teeth
• Type VI - rupture of cornea

Malignant Hyperthermia
Caused by Halothane , depolarizing muscle relaxants - succinylcholine
Tests
I. Creatine phosphokinase analysis
2. Muscle biopsy

• Loci CACNLIA 3 gene, Ryanodine receptor protein


• Rx cooling mechanism, .Dantrolene sodium
• Achondroplasia, hypochondroplasia, thanatophoric dwarfisn ~ mutation of fibroblast growth factor receptor, type
3 (FGFR 3) on chromosome 4pI6

Branchio Oto-Renal Syndrome

• Bronchial fistulas
• Anomalous ear pinna , preauricular pit, long narrow face, deepbite, hearing loss, renal dysplasia

Gorlin Syndrome I Basal Cell Nevus Syndrome


• Odontokeratocysts medulloblastoma
• Basal cell nevi lipomas and fibromas of skin, ribs
• Calcification offax cerebri
• Mutation of patched gene (PTe) on chromosome 9
• Skin cancer (Basal cell carcinoma)
• Not inherited
Facies of child with Gorlin syndrome .
• Widespread, down slanting palpebral fissures

Gardner Syndrome I Familial Polyposis


Dentigerous cyst, supernumerary teeth, delayed tooth eruption, osteomas.
sebaceous cysts
• Single central incisor : autosomal dominant inherited 7q36
• Cause holoprosencephaly, cebocephaly, mental retardation, death in infancy.

Treacher Collins Syndrome Sq32


Autosomal dominant, micrognathia, cleft palate, macrostomia, molar hypoplasia, down slanting palpebral fissure,
preauricular tags, deafness.
High Yield Facts .505

Cleidocranial Dysostosis
• Delayed eruption- of secondary teeth, supernumerary teeth, frequent caries, partial anodontia, malformed teeth or
roots.
• Wormian bones increased fracture macrocephaly, decreased stature hypoplasia of clavicle, high palate.
• Craniosynostosis syndrome FGR2gene
• Prematurely closed sutures
• Aperts ~ mitten like syndactyly of hands and feet (crouzons syndrome + syndactyly)
• Pierre Robin syndrome - cleft palate and micrognathia
• DiGeorge's - Thymic hypoplasia
Fibroblast growth factor receptor type 2 gene
• Velo-cardio-facial syndrome 22ql12
VSD .

Oculo Dento Digital Syndrome 6q


Microphthalmia, bent or fused 4th-5th fingers. Enamel-hypoplasia, wide alveolar ridge, wide mandible, small teeth,
gross dental caries.

Cystic Fibrosis
• Autosomal Recessive Deletion of Delta 508
• Tooth discolouration secondary to tetracycline use

Mucopolysaccharldoses
A. Hunter ~ X-linked recessive
B. Hurler
• Hereditary mental retardation - Fragile X syndrome

Ectodermal Dysplasia
• Vampire appearance/old man appearance
Anodontia/hypodontia
• Rx: A. Control body temperature
B. Antibiotics

Down's Syndrome
Scrotal tongue, increased periodontal diseased, decreased caries

Prader-Willi Syndrome
• Extreme obesity chromosome 15q11 .2
• Hypotonia, moderate mental retardation, almond shaped eyes, strabismus, scoliosis , enamel hypoplasia, dental
caries.

Angelman Syndrome
• Chromosome 15q 11 .2
Deleted maternal chromosome
Outbursts of laughter (Happy Puppet syndrome)

Beckwitb-Wiedemann Syndrome
• Over growth syndrome chromosome lip 155
• Macroglosia
.• Enamel-hypoplasia, prominent eyes, have Wilms' tumor.
506 • Dentogist: MCQs in Dentistry - Clinical Sciences

Williams' Syndrome
Friendly and talkative
I. Small body size
2. Aortaicstenosis
3. Wide mouth
4. Prominent blue eyes
5. Temper tantrums

MISCELLANEOUS

Primary incisors are more upright than permanent.


SNB and SNPg in children 78.1 and 77.4° Adults 80.0 and 83.0°
• ANB in children 4.9°
• Adults 2°
• Preschool years ~ 3-6 years ~ pre-operational period (Piaget)
• Preconceptual phase till 4 years
• Stage ofIntutive thought 4 - 7/8 years
• Aggression instumental-take others objects
Hostile - hurt others
• Oedipus or Electra complex - unusual feeling for the parent of opposite sex
• 3-6 years ~ Phallic stage
• A well-balanced profile in children ~ convex
• Lip posture assessed by draw ing an imaginary line from tip of nose to most anterior point on soft tissue chin lips
normally lie slightly behind this line .
• Lower lip is generally I mm anterior to this line
• Asymmetry usually manifests in lower facial 3rd
• Arch analysis best performed on diagnostic study models
• Developmental space - space present between primary teeth
• --Pi-is the most common missing tooth in primary dentition
• Majority of posterior crossbites are due to constr iction of maxillary arch
• Ankylosed primary dentition sequence from most common to least common

• 1E1D~-+&
• Normal growth curve is sigmoidal
• Pubertal growth spurt corresponds to steepest portion of the slope
• Pubertal growth spurt precedes menarche by more than I year .
• Time of adolescence is the period of preference for peer group interactions.
• Adolescence is a time of heightened caries activity due to increase intake of cariogenic substances and in attention
to oral hygiene procedures.
• By 5-10 years - neural and cranial growth almost completed
• Enamel of all permanent teeth is complete by 8 years
• Mandibular central incisor roots are complete by 9 years
• Pediatric films - N(). 0 size
• Occlusal films- No.2 size
• Snap-A-Ray (Rinn corporation, Elgin, IL) device has been used for film positioning
• Ideally rectangular collimation used
• Choice of restoration is a stainless steel crown rather than a 3 surface amalgam restoration in a decay prone person
• If numerous large lesions are present, they may be excavated and interim restoration placed
• It is best, if space maintenance can be implemented in 1st 6 months after extraction
• Children whose drinking water contain > 0.6 PPMF do not require any supplementation
• Topical fluoride play an important role in 3-6 years old
High Yield Facts _ 507

Topical Fluoride
Decreases the proportion of smooth-surface caries and corresponding increases in the proportion of pit and fissure
caries.
Most popular professional topical F agents ~ APF, NaF, APF ~ 1.23% F, ph 3.2
Applied semiannually - disposable polystyrene trays
• Recommended application time 4 min
• Systemic or professional topical Fluoride not recommended for Renal failure patients

Ca(OH)2

Catalyst paste Base paste


I . Ca(OH)2 1. Ca tungstate
2. ZnO 2. CaP0 4
3. Zn stearate in ethylene toulene 3. ZnO in glycol salicylate

• Ca(OHh softens under amalgam and composite resin

ZnOE Cement
Zno ....:; powder, liquid ~ eugenol
Zn acetate - accelerator
• Resin ~ increased fracture resistance
• To strengthen ZnOE cements, acrylic resin and alumina reinforcers are added.
• ZnP0 4 cement ~ cementation of stainless steel crowns

GIC - Physicochemical Bond to Tooth Structure


Ca Aluminosilicate glass mixed with polyacrylic acid
• GlC binds to dentin by hydrophilic carboxyl groups
• Tartaric acid ~ accelerate the rate of hardening without decreasing working time
• Itaconic acid - increased the reactivity of polyacrylic 'acid to glass
• Polymaleic acid - modify the reaction
• GIChydrous
Anhydrous has dehydrated polyacrylic acid
• GlC has co-efficient of thermal expansion similar to that of tooth structure
• Purpose of cav ity varnishes is to reduce microleakage at restore margins and inhibit penetration of corrosion
products from amalgam into dentin.
• Compomers - polyacid -modified resin composite Fluoride containing glass filler and polyacid must be heat cured
• Resin modified GIC
• 3 chief products
I . Fuji II LC (GC)
2. Vitremer
3. Photac - fil (ESPE)
• Most ideal dentin replacement material ever produced

Dentin Bonding Agents (DBA)


• Ist generation ~ holophosphorus esters of BIS GMA
• 2nd ~ polyurethanes
• 4-META
• Removing smear layer increases the effectiveness of DBA
• 4-MET A - 4 methacryloxy ethyl trimellitic anhydride
508 • Dentogist: MCQs in Dentistry - Clinical Sciences

Amalgam
• Material of choice for Class 1 and II restorations
• Cu6Sn5 matrix decreases corrosion of tin, prevents secondary weakening of restoration.
• Zn in Amalgam -t Scavanger for O 2
• ADA requires that no more than 20 microns/em of expansion/contraction after 24 hours
• ADA requires that amalgam have maxi 5% creep
• Gamma2 phase is most susceptible to corrosion
• Final polish of amalgam -t: tin-oxide compound
• Polished after 24 hours
• Cermet cement harden initially in about 4-5 min

Composite
BIS GMA+TEG DMA
Produces desired handling qualities
Composite resin
Benzoyl peroxide - initiator
Tertiary amine - catalyst
Visible light - activated composite resins contain a
Diketone initiator -t camphoroquinone
Amine catalyst - dimethyl aminoethyl methacrylate
Wave length 470 nrn
Advantages of light activated composite resins
A. Ease of manipulation
B. Control of polymerization
C. Lack of need for mixing
D. Less chance of air incorporated
• Mechanism of composite wear is due to loss of resin matrix
• Microfilled composite resin -t easily polishable
• Decrease filler results , in decrease setting time, increase in wear
• Macrofilled composite resin -t increase wear resistance
• Hybrid composites - in stress bearing areas

Cavity Preparation
• GV Black - Steps for cavity preparation 1924
All instrumentation for restorative procedures -t use high speed air turbine handpiece (1,00 ,000-3,00,000 rpm)
combined with coolant. Coolant may be water spray/air.
Water spray coolant recommended for high speed instrumentation.
Water spray to remove old amalgam restoration or using diamond burs.
Slow speed handpiece 500-15000 rpm - used for caries removal, polishing and finishing procedures.
Hand instrumentation for final caries removal or planing enamel walls.
Air abrasion uses a stream of purified Aluminium Oxide particles 27-50 micrones- forced under 40-120 psi
introduced by Black in 1945.
Mesiobuccal pulp hom is most prominent in primary teeth.
Enamel rods ofgingival 3rd of crown extend in an occlusal direction from DEI in primary teeth.
Primary teeth have broad, flat, proximal contact areas.
Primary teeth have narrow occlusal surfaces compared to secondary teeth.
High Yield Facts _ 509

Rubber Dam
Introduced by S. C. Barnum.
5 x 5" medium gauge rubber dam best suited for use in children.
Rubber dam frame ~ Handidam, Aseptico, Kirkland, WA Young's, Nyguard ostby.
For single Class I or Class V restorations, only the tooth being restored need to be isolated.
Interproximal lesion ~ isolate atleast one tooth anterior and one teeth posterior to the tooth being restored .
Slit tech ~ used to isolate more teeth.
Used for isolation of maxillary quadrant.
12-18" dental floss-used to retrieve clamp .
Ist cut interproximal septa ~ to remove Rubber dam.

Class I cavity in Primary Teeth


Primary molars ~ most caries susceptible primary teeth.
Ideal pulpal floor depth is 0.5 mm into dentin .
Length of cutting end of No. 330 bur is 1.5 mm.
Isthmus be 1/3rd the intercuspal width .
Buccal and Lingual walls converge.
Mesial and Distal walls flare at marginal ridge.
Placement of bases in Primary teeth is uncommon
Burnish the carved amalgam when the amalgam has begun its initial set.

Class II cavity in primary teeth


Proximal box be broader at cervical than occlusal aspect. Buccal and Lingual walls create 90 0 angle with enamel.
Gingival wall be flat - not beveled.
Axial wall be 0.5 mm into dentin.
Axiopulpalline angle rounded.
Mesiodistal width of gingival seat = I mm.
In preschool children with large proximal carious lesions, stainless steel crowns preferred to amalgam because of
durability.
Pins for retention in primary teeth contraindicated.
Multiple matrices placed with T-bands.
Sectional matrices ~ 1/2 inch long-fit in prepared proximal areas.
Spot welded matrix:
Matrix band be atleast I mm higher than marginal ridge of adjacent tooth.
Restore adjacent interproximal lesions simultaneously.
Rubber abrasives used for final polishing.
Primary enamel is approximately one half the thickness of permanent enamel.
Sealants are not widely used on primary teeth because of two reasons :
I. difficulty in moisture control.
2. limited life span of tooth .
Sealant retention on primary molars is similar to that of retention on permanent molars.
95% after I year
93% after 3 years.

Preventive Resin Restorations (PRR)


3 types ofPRR ~ Type A,B,C
Type A or 1 ~ most conservative tooth prep
Type 1 PRR - when pit and fissure decay is minimal -sealant.
Type 2 PRR - extends into dentin ~ wear resistant composite.
Type 3 PRR - extends into dentin, wear resistant resin used to restore cavity preparation then apply pit and fissure
sealant.
510 • Dentogist: MCQs in Dentistry - Clinical Sciences

Leinfedler ~ Class II preparation restricted to regions of caries .

Pre-trimmed crowns-Unitek stainless steel crowns, 3M Company, St. Paul, MinnesotaandDenovo crowns, Denovoco,
Arcadia, CA. .
Pre-contoured crowns - Nicr Ion crowns, Unitek SS crowns, 3M Company, St. Paul, MN.

Tooth Preparation for Stainless Steel Crowns

• Preformed or stainless steel crowns introduced to pediatric dentistry by Humphrey in 1950.


• Extension outline of class I restoration is determined by extent of caries, undercuts created with No.35 inverted
cone bur or No.1/2 round bur ~ at gingivo axial and incisoaxialline angles .
• Occlusal ~ 1.0-1.5 mm gingival proximal margin have a feather-edge finish line occlusobuccal and occlusolingual
line angles rounded.
• Size 4 crown ~ most frequently used for molars.
• Seat the Lingual first and apply force in buccal direction.
• Properly trimmed crown extends approximately I mm into gingival sulcus.
• Contour and crimp the crown to form a tight fitting crown .
• Contouring with no.114 ball and socket pliers or with no.l37 gordon Pliers.
• Crimped with No. 137 plierslNo . 800-417.
• Final smoothing and polishing of crown margin done before cementation.
• Crown filled 2/3rds with cement.
• Angles etch resin crown - most esthetic in anteriors and frequently placed crown.
• Preveneered stainless steel crowns ~ Kinder KrownsMayelin dental, studios, Minneapolis, New Smile primary
crowns .
• ART promoted by WHO
• Calcitraumatic band represents an interruption of opposition between secondary and tertiary dentin.
• Belanger suggests that percussion should be done gently with tip of a finger and not with the end of dental mirror.
• Pulp Polyp
Rx coronal pulp amputation and formacresol pulpotomy
• Rationale for indirect pulp capine (IPC) is that few viable bacteria remain in deeper dentin layers, after sealing the
cavity, they are inactivated.
Ultimate objectives of IPC
1. Arrest carious process
2. Promote dentin sclerosis
3. Stimulate tertiary dentin formaiton
4. Remineralize carious dentin
Direct pulp capping - carried out when healthy pulp is inadvertently exposed during operative procedure
~ internal resorption - failure of DPC .

Pulpotomy
Most commonly used formacresol (Buckley 's solution)
Buekley's solution ~ . 1. Formaldehyde
2. Cresol
3. Glycerol
4. Water
Cotton pellet moistened with Buckley's solution placed over pulp stumps for 5 min.
Fixation coronal 1/3rd, inflamed middle 3rd,vital tissue apical 3rd
Glutaraldehyde - mild fixative and is potentially less toxicanine
Pulpectomy indicated in teeth that show evidence of chronic inflammation or necrosis in radicular pulp.
Goal of pulpectomy is to maintain primary teeth
High Yield Facts _ 511

• Root Canal Fillings


RCF materials ~ ZOE paste , iodoform paste,
Ca(OH)2 not generally used because of internal resorption.
KRI paste:
,1.
Mixture of iodoform, camphor, parachlorophenol, menthol.
MAISTO paste :
,1.
• KRI paste components + Zno, thymol, lanolin
Ca(OH) 2 + iodoform mixture
• Vitapex, Neo dental chemical products
Ca(OH)2 + iodoform
Endoflas ~ US

Patient Management by Domain


Physical Domain - Hysterical Children Pappose Board
Pharmacological Domain
Aversive - HOM
Reward - oriented Domain:
Linguistic domain ~ most reasonable
Ontology means our way of being
Ist appointment - examination, prophylaxis, fluoride treatment
Voice control can be used in deaf children
HOM - A. hysterically crying child
B. legitimate technique
Indicated in normal child who is old enough to understand directions of denti st
55% impact ~ nonverbal expressions
Dentist must not lie about needle.
Children under 36-40 months of age often behave better if parent accomplish them
Acquired fears ~ told to child
. Learned fears - due to previous difficult/painful experience.

Misbehaving Children
Category I - emotionally compromised
Category II - shy, introverted child
Category III - frightened child
Category IV - who is adverse to authority

Gingivitis
Gingivitis occurs in 1/2 of the population by age 4 or 5 years 100% at puberty ~ due to increased steroid hormones .
Peak prevalence of puberty gingivitis is 10 years in girls, 13 years in boys.
Drug induced gingival enlargement ~ also affected by oral hygiene, painless, fibrous, firm, pale pink, little tendency to
bleed, enlargement occurs Ist in interdental region.
Rx: Gingivectomy/flap with internal bevel.
• Strippling
Loss of attachment and recession that occurs with labially malposed tooth contributed by smokeless tobacco, habit
related.

Adult Onset Periodontitis- 14-17 Years


UP ~ neutrophil chemotactic defect
A. comitans
Rx: Systemic tetracycline
Metronidazole ± Amox
512 • Dentogist: MCQs in Dentistry - Clinical Sciences

GJP
Not self limiting heavy plaque and calculus found
Rx: Local therapy + systemic antibiotics
Localised prepubertal periodontitis
Rx: A. local debridement
B. tetracyclines
DOC: metronidazole

NUGINUP (Necrotizing Ulcerative GingivitislNecrotizing Ulcerative Periodontitis)


In late teens and early twenties
Malnutrition, viral infection, stress, lack of sleep
Increased spirochetes, P intermedia
Rx: Local debridement
Penicillin/metronidazole

Hypophosphatasia
Decreased alkaline phosphatase
(Hypophosphatemia decreased phosphate in blood)
Premature loss of dec iduous teeth ~ severe bone abnormalities
Neonatal death
Early loss of primary teeth ~ only clinical sign
Defective cementum formation
Dental prognosis for secondary teeth is good

Leukocyte Adhesion Deficiency/Generalised Prepubertal Periodontitis


Rare
Surfaceglycoproteins on leukocytes are defective
Poor migration, decreased phagocytosis.

Histiocytosis 'X' / Langerhans cell Histiocytosis


Diagnosis biopsy
Rx: Radiation and surgery
Systemic chemotherapy for disseminated cases.

Acute Lymphocytic Leukemia


ALL most common in children and has best prognosis, gingival enlargement.
AML has poor long-term survival rate . .

Space Management
• Missing Primary Teeth are Replaced for
Space maintenance
Function
Speech
Esthetics
Loss of primary canine as a result of either trauma/decay is rare .
\
Band and loop ~ maintain space of single tooth.
Indications: Unilateral loss of primary 1st molar before/after eruption of 6
Bilateral loss of primary molars before the eruption of permanent incisors.
Band is I mm below mesial and distal marginal ridge, loop 36 mm wire, Buccolingual dimension of loop = 8 mm.
High Yield Facts _ 513

• Lingual Arch
When teeth are lost in both quadrants ofsame arch
• Distal shoe appliance:
Contraindications -
a. medically compromised patients
b. SABE patients
Space maintenance not required for missing primary incisors

Crossbite
Rx: A. W arch ~ 36 mil wire 1··1.5 mm off palate
B. Quad helix-ershort wire-e tooth to be moved)
Quad helix- To correct post crossbite, discourage finger habit
Wire spring activated 2 nun/month
Jack screw I tum/day 0.25 mm activation
• LA (Cont.)
Benzocaine topical anesthetic - good
Effective in 30 sec - 5 min
Amides LA are more frequently used because of decreased allergy
Methyl paraben - preservative
Drug Maxi dose mglkg
I. Lidocaine 2% 4.4. (300 mg max)
2. Mepivacaine 2% - levonor demn 4.4, (300 mg)
2% = 20 mglml x 1.8 ml / cartridge = 36 mg
• Needles
short 20 mm
long 32 mm 30 mm gauge needle used for most intraoral injection in children.
Extra short 10 mm - 30 gauge needle - maxillary anterior injection
Mandibular foramen is at/slightly above occlusal plane during the period of primary dentition
In adults - 7 mm above occlusal plane
• Pediatric Extraction Forceps
No. 150s, 151s
Forceps - seat Lingual/Palatal beak Ist then rotating the facial beak into proper position
l) No. 301 straight elevator used to luxate maxillary molars.
2) Extraction -150s
Anterior teeth rotation movement No.1 forceps
3) Mandibular molars - No. 151s forceps . Buccal-Lingual motions
Mandibular canines, premolars- Rotational movements
Soaking gut sutures prior to use in glycerin softens them
Intraoral silk suture be removed at 5-7 days

Piaget
Sensorimotor- birth - 18 m
Preoperational 18 months - 7 years
Concrete- operational 7-12 years
Formal - operational 12 years and onward
Child in transitional years masters the emotional abili ty to delay'gratification
• Lower lip normally covers 1-2 mm of maxillary incisors at rest
• Odontomas interfere with normal tooth eruption
• Ectopic eruption is most often associated with permanent maxillary first molar and mandibular lateral incisors.
Diastema> 3 mm ~ due to mesiodens
514 • Dentogist: MCQs in Dentistry - Clinical Sciences

MODEL ANALYSIS

Tooth size analysis - Bolton - 1958


Most accurate (modification of Hixon old father analysis)
Hixon old father analysis 1984 uses lower incisors widths and width of unerupted premolars measured from radiographs
to predict permanent tooth size
Tanaka Johnston Analysis
Vl Most clinically useful
U
''::; Requires no additional X-rays/Tables
c:
o Total facial height, normally measured from nasion to menton
"'C
o Upper facial height 45%, lower facial height 55%
"'C
Q.) • Ricketts Eline:
a..
Line connecting tip of nose with the anter ior contous of chin, upper lip Imrn behind the line and lower on or
slightly behind the line.
• Caphalometric ~ 3 superimpositions made
Sella nasion line, anterior cranial base
Maxilla, Pterygomaxillary fissure, zygomatic process
Mandibular ~ inner surface of mandibular symphysis, outline of mandibular canal, unerupted 3rd molar crypts
• Permanent 2nd premolars evident at 4 years on X-ray

Dental caries
• Females had slightly higher DMFS scores than males
Halo effect - increases in distribution of foods and beverages prepared with fluoridated water.
Sugar - arch criminal of dental caries (Newbrun 1969)
Vipeholm study - Gustafson 1954
Hopewood House - New South Wales, Australia
Sullivan and Goldsworthy 1958, Sullivan and Harris 1958
Phosphates decreases caries
Foods high in fat, protein, F, Ca protect against dental caries
Fats ~ coat the teeth
i. Reduce sugar retention
ii. Decreases sugar solubility
iii. Toxic to oral bacteria
Tarmins and cocoa ~ suppress dental caries '
• Tooth paste contain NaF or 'N'amono Fluora Phosphate as acti ve ingredient
. FDA 1100 PPM
Most popular Fluoride mouth rinse NaF
Brushing after all meals is ideal
Thorough cleaning of teeth and massaging gingiva before bed with additional brushing after breakfast and after
evening meal.
Caries involving occlusal surfaces account for almost 60% of total caries experience.
Occlusal surface account for 12.5% of total number of tooth surfaces.
Interproximal caries more prominent in primary dentition.
Water fluoridation reduces interproximal caries in primary teeth by 45%.
Occlusal caries in fluoridated community decreases by 23% and Buccal and Lingual caries reduced by 32%.
90% of all lesions in 1st permanent molars-pit and fissure caries.
Increased incidence in occlusal caries greatest during Ist four years .after eruption.
Shallow, wide U-shaped fissures - self cleansing, caries - resistant.
Deep narrow I-shaped fissurecaries eptible.
Fissure walls are Ist affected by caries.
3-4 years may be required for dentinal involvement to occur.
Hyatt's Prophylactic Odontotomy 1924
High Yield Facts _ 515

Bodecker - 1929 clean fissure with explorer and flow a thin mix of oxy P04 cement
• Acid etching - Buonocore 1955
UV light A. 365 nm (visible light wave length 470 nm)
Photoactivated resins use a diketone initiator such as camphoroquinone and reducing agent tertiary amine to initiate
polymerization.
Dia Methacraylatic Monomors (DMA) possess estrogen like effects on cultures of breast cancer tumor cells.

Diagnosis of Caries
J. Catch of explorer
2. Softness at the base of the area
3. Opacity / loss of normal translucency
4. Softened enamel adjacent to pit and fissure
5. Electronic caries detection found to have high sensivity (0.7-1) and high specificity (0.7-0.96) .

Preventive Resin Restoration


PRR ~ Simonsen 1978
ART - minimal- intervention technique touted by WHO
Complete retention rate varied from 92% after 1 year to 28% after 15 years
ART performed in Thailand, Zimbabwe
Acid etching.
Prosphoric acid 35-40% for 15-60 sec for 10 and 2 0 teeth
20 sec ~ permanent teeth
30 sec ~ primary teeth
Etched zone 10-30 micro meter depth
Qualitative porous zone - 20 microns.
Quantitative porous zone - 20 microns

Etching Pattern
Type I - lost prism cores
Type 2 - prism peripheries lost
Type 3 - general surface roughening and porosity with no exposure of prism cores / peripheries
Resin tags penetrate etched enamel to a depth of 25-50 microns
1. Provide mechanical retention
2. Surround enamel crystals and provide resistance to demineralizatio
Normal physiologic root closure of permanent teeth may take 2-3 years after tooth's eruption, deciduous teeth may
take I Y2 to 2 years.
Ca(OHh, routinely placed over dentin in any area that is appreciably deeper than DEJ - to protect dentinal tubules,
Odontoblasts, pulp by sealing dentin and protecting pulp through sclerosis of dentinal tubules .
IRM over Ca(OHh arrest the progression in a caries - active mouth, promote favourable pulpal response.
Indirect Pulp Capping (IPC)
Goal is to promote pulpal healing by removing majority of infective bacteria and by sealing the lesion, which
stimulates sclerosis of dentin and reparative dentin formation
Direct Pulp Capping (DPC)
<2 mm size

Apexogenesis
Goal is to maintain radicular pulp's viability to allow apexogenesis/apical closure. Used to save young permanent teeth
. with exposed vital/infected coronal pulp .
Apexification used to promote root elongation or calcific root closure across enlarged apex of tooth.
Forms porous osteodentin/cementoid bridge.
Reviewing radiographs 1 month after injury detects signs of pulpal necrosis and inflammatory resorption.
At 2 months, replacement resorption detected.
Cvek's partial pulpotomy by removing 2 mm of pulp .
516 • Dentogist: MCQs in Dentistry - Clinical Sciences

Apical Barrier Technique Harbert 1991


Employs generic tricalcium P0 4 powder condensed through root canal.
To the apex. Gutta-percha then packed against TCP plugs at the same visit.

Trauma
Lat. luxation - splint for 3-8 weeks
Extrusion - (Splint) 2-3 weeks
Avulsed tooth 1-2 weeks
Root fracture 2-3 months
Best transport medium for avulsed teeth is cell culture media such as viaspan or Hanks balanced salt solution (HBSS)
(Save-A-tooth) (trade name)
Other transport media are
Milk-best alternative
Saliva
Water -+ PDL cells swell and rupture
Ortho wire used for splinting/30-60 pound

Rapid Palatal Expansion (RPE)


Teeth opened I mm/day to deliver 2000 to 3000 g of force.
Active Rx takes 10-14 days .
• Slow Palatal Expansion
900-1300 g force

Orthopedic Appliances

• Class II malocclusion with maxillary protrusion -+ Head gear with 12-16 ounces/side for 12-14 hours.
• Class III with maxillary deficiency -+ Reverse pull headgear or face mask
• Class III with mandibular excess -+ Chin cup therapy.
• Vertical maxillary excess -+ high pull head gear
• Vertical deficiency -+ Head gear with junctional appliances
Space maintenance undertaken unless the tooth is expected to erupt within 6 months
6 months time be anticipated for every mm of bone that covers the perm . tooth.

Growth period
Puberty - individual. becomes capable of sexual reproduction.
14 years for boys 12 years for girls
Female menarche serves as a signal that growth is ending .
Growth spurt in females peaks at 9 em change/year at age 12. In Males, it peaks at just over 10 em at age 14.
Vertical maxillary height growth is often greater in females ,
Roots of all teeth are considered to have completed by age 16 except for those of 3rd molars (25 years).
Adolescents are passionate, irascible, apt to be carried away by their impulses - Aristotle.
Adolescence represents the final transition socially from childhood to adulthood.
Early adolescence 12-14 years
Late adolescence 16-21 years
High Yield Facts _ 517

Fluoride

Water fluoridation-most cost effective means of caries prevention.


Dental mottling Ist reported by Kuhns in 1888.
Denti di chiaie - due to volcanic fumes or subterrarean fumes .
Denti neri - black teeth
Denti seritti - teeth written upon
Colorado brown stain - Dr. Frederick s Mekay. .
1926 Mc Kay openly implicated drinking water supplies
Dean 1934 gave classi fications of fluorosis
• Once tobacco use is discontinued, leukoplastic lesions usually disappear within a week/two
• Bulimia - those who vomit regularly to purge themselves of food in a misdirected attempt to control their weight.
• Perimyolysis/Erosion
Bulimia
Pulpal pathosis, elongated clinical crowns, gingival recession, loss of vertical dimension.
• Females using oral contraceptive and smokers run a high risk of post-surgical dry socket.
Full mouth survey 21 films
Camouflage is the orthodontic movement of teeth without changing the underlying skeletal malocclusion and
considered direct only when soft tissue profile is acceptable.
Of Class II women
Class III - men
Uprighting spring bent from 19 to 25 mil stainless steel wire.
Force of activation should provide approximately 75-100 g force to upright molar
Bruxer have sore masseter
Child should be able to open approximately 35-45 mm
20-35 mm ---t muscular problem < 20 mm - joint problem
Lat. movement 8-12 mm

Steps in composite restorations


Base/liner placed on exposed dentin
Etch for 15 sec
Rinse 5 sec
Primer applied 15 sec
Dried with stream of air
Adhesive resin applied light cured for 10 seconds
Composite resin applied
Finishing polishing
Hypoplastic spots
Enamel microabrasion and restoring with composites.
composite resin/porcelain veneers chamfer finish line.

Primer facilitates penetration of an adhesive resin into demineralized dentin and enamel to form a hybridized layer of
resin/tooth.
Amber filters are used with light curing unit
Proper value (Munsell whiteness) determined by squinting (squint test).
Resensitize eyes by staring momentarily at dark blue or gray object.
Primary retentive feature is beveled Enamel cavosurface margin minimum of 1-2 mm length. Beveling allows maximal
bond strength and minimizes leakageby exposing the ends of Enamel rods by etching.
518 • Dentogist: MCQs in Dentistry - Clinical Sciences

Bleaching

Power bleach ~ concentrated H202 solution applied while heating the teeth with electric lamp .
Night guard bleach - 10% carbamide peroxide worn at night for 2-3 weeks
Types of mouth guards
I - stock - inexpensive-bulky-least retentive
II - mouth formed - good retention
lj III - custom fabricated - most comfortable
"=c::
.g (Ex.) - polyvinylacetate polyethylene
.g mouth formed mouth guards
8:. thermoplastic variety - (Boil and bite technique).

You might also like